Colon Flashcards

You may prefer our related Brainscape-certified flashcards:
1
Q

Which of the following favors appendiceal carcinoid over appendiceal adenocarcinoma?

A. Bulbous swelling involving the distal third of the appendix

B. Slow growth

C. Direct extension into caecum

D. Ileocecal lymph node enlargement

E. Periappendical fat standing

A

A. Bulbous swelling involving the distal third of the appendix

Carcinoid often appears as a bulbous swelling of the distal third of the appendix, in contrast to adenocarcinoma which tends to involve the proximal third and causes diffuse thickening of the appendiceal wall.

How well did you know this?
1
Not at all
2
3
4
5
Perfectly
2
Q

A 60-year-old woman presents with abdominal pain and diarrhoea six weeks post-hip replacement surgery, with her recovery being complicated by a hospital-acquired pneumonia. CT is performed. When considering a diagnosis of pseudomembranous colitis (PMC) which is the most common finding?

A. Intense mucosal enhancement

B. Enlarged peri-colic blood vessels

C. An irregular or discontinuous mucosal lining

D. Pericolonic fat stranding

E. Colonic wall thickening

A

E. Colonic wall thickening

Although all of the above are recognized findings in PMC, colonic wall thickening from minor to gross thickening is the most common feature.

The relatively mild pericolonic stranding with marked colonic wall thickening may help to distinguish PMC from other colonic pathologies.

How well did you know this?
1
Not at all
2
3
4
5
Perfectly
3
Q

Which of the following most favors Crohn’s versus pseudomembranous colitis?

A. Ascites

B. Absence of small bowel involvement

C. Fibro fatty mesenteric proliferation around involved colon

D. Colonic wall thickening of 11mm

E. Low attenuation mural thickening (accordion sign)

A

C. Fibro fatty mesenteric proliferation around involved colon

Fibro fatty mesenteric proliferation around involved colon is more likely to indicate crohn’s disease.

How well did you know this?
1
Not at all
2
3
4
5
Perfectly
4
Q

Which of the following frequencies would most help in the differentiation between recurrent tumour and fibrosis in a 62-year-old with prior chemoradiotherapy for low rectal cancer?

A. Axial and coronal balanced gradient echo

B. Sagittal T2 fat suppressed fat-spin echo (FSE)

C. Axial TIW fat suppressed FSE before and after gadolinium

D. Sagittal TIW fat suppressed FSE post gadolinium

E. Coronal TIW fat suppressed FSE post gadolinium

A

C. Axial TIW fat suppressed FSE before and after gadolinium

May help differentiate tumour from fibrosis and improves destruction of tumour spread to adjacent structures/vessels.

Axial and coronal balanced gradient echo are rapidly acquired breath hold sequences providing anatomical overviews.

Granulation tissue, hematoma and radiation-induced inflammatory change can all display SI similar to tumour.

Peripheral enhancement with central necrosis is commonly seen in recurrent disease.

How well did you know this?
1
Not at all
2
3
4
5
Perfectly
5
Q

A 70-year-old man with sudden onset left-sided abdominal pain underwent evaluation with CT. Fat stranding and colonic wall thickening is noted in the distal descending/ proximal sigmoid colon. Diverticulae are noted. Which feature is most likely to represent a colonic tumour rather than diverticulitis?

A. Colonic wall thickening

B. Pericolonic fat stranding

C. Fluid in the sigmoid mesentery

D. Engorgement of the mesenteric vessels

E. Increased permeability on CT perfusion

A

E. Increased permeability on CT perfusion

Increased blood volume, increased blood flow, decreased mean transit time and increased permeability are parameters on CT perfusion which have been shown to favor a diagnosis of cancer rather than diverticulitis. C and D have high positive predictive values in diagnosing diverticulitis and A and B occur in both.

How well did you know this?
1
Not at all
2
3
4
5
Perfectly
6
Q

A usually fit and well 30-year-old female presents with left-sided abdominal pain. Normal uterus and ovaries are identified. In the left iliac fossa there is a 4cm hyperechoic mass which is non-compressible between the colon and abdominal wall. No colonic wall changes are identified. There is no free fluid. The lesion is surrounded by a hyperechoic border with no Doppler flow. Which is the most likely diagnosis?

A. Epiploic appendigitis

B. Diverticulitis

C. Omental infarction

D. Omental metastases

E. Mesenteric panniculitis

A

A. Epiploic appendigitis

Self-limiting condition with fat density mass 1-4cm between colon and abdominal wall.

The absence of blood flow on Doppler due to torsion helps distinguish from acute diverticulitis.

Omental infarcts are usually large, right-sided and cake-like in appearance.

How well did you know this?
1
Not at all
2
3
4
5
Perfectly
7
Q

Which is the most common source of false positives in cathartically prepared CT colonography by Computer aided detection (CAD)?

A. Haustral folds

B. Untagged/poorly tagged stool

C. Ileocecal valve

D. Electronic cleansing and tagging artefact

E. Extrinsic compression

A

A. Haustral folds

Most common source of FP in CAD in cathartically prepared CT colonography.

Untagged/poorly tagged stool is the second most common FP in cathartically prepared and most common source of FP in non-cathartically prepared CT colonography.

How well did you know this?
1
Not at all
2
3
4
5
Perfectly
8
Q

A 37-year-old man on the intensive care unit, 3 weeks post-operatively for complicated small bowel resection for Crohn’s disease, is noted to have an oozing from a site on his anterior abdominal wall. His renal function remains normal and inflammatory markers are persistently elevated with White Cell Count (WCC) 15.9 and C-reactive Protein (CRP) 124. On a CT of the abdomen 5 days ago, no collection was demonstrated. CT fistulography is being considered. Which is the best answer with regards to CT fistulography?

A. Conventional fistulography has a higher spatial resolution

B. Iodinated contrast should be used, diluted 1 in 100

C. Oral contrast is helpful in most cases

D. Intravenous (IV) contrast should be avoided

E. CT fistulography has a greater temporal resolution

A

A. Conventional fistulography has a higher spatial resolution

CTF helps to delineate fistulous tracks in critically ill patients where prolonged contrast studies are not feasible.

Iodinated contrast (300mg iodine/ml) should be diluted 1 in 10, with a volume based on an estimate of the length of bowel or volume of cavity being investigated.

Oral contrast can confuse the origin of intraluminal contrast and is best avoided, whereas IV contrast is helpful.

Conventional fistulography has a higher spatial and temporal resolution.

How well did you know this?
1
Not at all
2
3
4
5
Perfectly
9
Q

With regards to MRI in patients being investigated for Crohn’s disease, which of the following is the least desirable characteristic of ultrafast sequences based on steady state precession?

A. A high sensitivity to motion artefact

B. The time taken to acquire an image

C. The degree of contrast between bowel wall and lumen

D. Degree of soft-tissue differentiation when compared to other sequences

E. A black boundary effect

A

E. A black boundary effect

These sequences have become the main MR technique for imaging small and large bowel in Crohn’s disease in recent years.

Uniform luminal opacification is obtained with high contrast between bowel wall, lumen and mesentery.

The sequences are relatively insensitive to motion artefact.

Black boundary artefact can obscure small lesions on the bowel wall, although fat-suppression can reduce the artefact.

How well did you know this?
1
Not at all
2
3
4
5
Perfectly
10
Q

In the MR staging of rectal tumours, which of the following is the single best answer?

A. A distance of less than 3mm from the mesorectal fascia is most likely to reflect circumferential resection margin (CRM)

B. There is a prognostic difference between different types of T3 tumours depending on tumour size

C. Untreated mucinous and non-mucinous tumours are difficult to distinguish on T2 sequences

D. Untreated tumour signal is similar to muscle on T2

E. Non-mucinous tumour signal intensity is similar to fat

A

B. There is a prognostic difference between different types of T3 tumours depending on tumour size

The MERCURY Trial showed patients with T3 disease with more than 5mm of extramural spread have a markedly worse prognosis than those with less than 5mm spread beyond the outer muscle layer.

A distance less than 2mm from mesorectal fascia indicated Circumferential resection margin (CRM) involvement.

Tumour signal is usually in between SI of muscle and fat.

Mucinous tumour is higher signal than non-mucinous which is higher than fat on T2.

How well did you know this?
1
Not at all
2
3
4
5
Perfectly
11
Q

A 37-year-old man on the intensive care unit, 3 weeks post-operatively for complicated small bowel resection for Crohn’s disease, is noted to have an oozing from a site on his anterior abdominal wall. His renal function remains normal and inflammatory markers are persistently elevated with White Cell Count (WCC) 15.9 and C - reactive protein (CRP) 124. On a CT of the abdomen 5 days ago, no collection was demonstrated. CT fistulography is being considered. Which is the best answer with regards to CT fistulography?

A. Iodinated contrast should be used, diluted 1 in 100

B. Oral contrast is helpful in most cases

C. Intravenous (IV) contrast should be avoided

D. Conventional fistulography has a higher spatial resolution

E. CT fistulography has a greater temporal resolution

A

D. Conventional fistulography has a higher spatial resolution

CTF helps to delineate fistulous tracks in critically ill patients where prolonged contrast studies are not feasible.

Iodinated contrast (300mg iodine/ml) should be diluted 1 in 10, with a volume based on an estimate of the length of bowel or volume of cavity being investigated.

Oral contrast can confuse the origin of intraluminal contrast and is best avoided, whereas IV contrast is helpful.

Conventional fistulography has a higher spatial and temporal resolution.

How well did you know this?
1
Not at all
2
3
4
5
Perfectly
12
Q
  1. A 70-year-old hospitalized male patient presents with watery diarrhoea and abdominal pain. CT of the abdomen demonstrates marked circumferential bowel wall thickening involving the entire colon, with minimal pericolonic stranding and a small amount of ascites. The small bowel appears normal. What is the most likely diagnosis?

A. Crohn’s disease

B. ischaemic colitis

C. diverticulitis

D. pseudomembranous colitis

E. ulcerative colitis

A

D. pseudomembranous colitis

Pseudomembranous colitis is an acute infectious colitis caused by Clostridium difficile and its toxins A and B; this pathogen has become increasingly common largely due to widespread use of broad-spectrum antibiotics.

The commonest CT finding is of colonic wall thickening (due to mural edema and the presence of pseudomembranous), which is typically greater than in other causes of colitis apart from Crohn’s disease. Pericolonic inflammatory changes are disproportionately mild relative to the marked wall thickening. Ascites is common, and this, together with the lack of small bowel involvement, can help to distinguish pseudomembranous colitis from Crohn’s colitis.

Ischaemic colitis demonstrates a lesser degree of wall thickening, and is usually segmental, tending to affect the watershed areas of the colon.

How well did you know this?
1
Not at all
2
3
4
5
Perfectly
13
Q
  1. A 34-year-old man presents with acute left lower quadrant pain following unaccustomed exercise. CT of the abdomen demonstrates a 2.5 cm oval lesion with attenuation value of –60 HU abutting the sigmoid colon, with surrounding inflammatory changes. The sigmoid colon itself appears normal. What is the most likely diagnosis?

A. omental infarction

B. diverticulitis

C. epiploic appendagitis

D. liposarcoma

E. appendicitis

A

C. epiploic appendagitis

Acute epiploic appendagitis is thought to result from torsion of one of the fatty epiploic appendages arising from the serosal surface of the colon.

It usually occurs in young men, presenting as acute lower quadrant pain, and is associated with obesity and unaccustomed exercise.

Typical CT findings are of an oval pericolonic fat density lesion of<5 cm, with surrounding inflammatory changes, most commonly in the sigmoid, descending or right hemi colon.

Right-sided epiploic appendagitis may be mistaken clinically for appendicitis.

Omental infarction typically appears as a larger, heterogeneous lesion, usually affecting the caecum or ascending colon.

Acute diverticulitis usually occurs in older patients, and is associated with colonic diverticula and wall thickening.

Liposarcoma is rare, but is included in the differential of a fat-containing intra-abdominal mass.

How well did you know this?
1
Not at all
2
3
4
5
Perfectly
14
Q
  1. What is the primary imaging investigation for staging of colon cancer diagnosed at colonoscopy?

A. CT of the abdomen and pelvis

B. CT of the thorax, abdomen and pelvis

C. abdominal ultrasound scan

D. double-contrast barium enema

E. 18FDG PET/CT

A

B. CT of the thorax, abdomen and pelvis

Patients with colon cancer diagnosed endoscopically or suspected following barium enema should be imaged for staging purposes. Objectives of staging include determination of size and local extent of tumour, assessment of the extension of tumour into adjacent structures, and detection of local and distant nodal involvement and presence of metastatic disease. Abdominal ultrasound scan alone is not considered sufficient, and CT of the chest, abdomen and pelvis with oral and intravenous iodinated contrast medium is the primary imaging investigation. The liver is the commonest site of distant metastases, but pulmonary metastases occur in 5–50% of patients. 18FDG PET/CT is not used in initial staging but is particularly useful for detecting recurrent disease.

How well did you know this?
1
Not at all
2
3
4
5
Perfectly
15
Q
  1. A 78-year-old man presents with abdominal pain. A plain abdominal radiograph demonstrates a distended, inverted U-shaped loop of bowel devoid of haustra, extending from the left iliac fossa inferiorly to just beneath the left hemidiaphragm superiorly. What is the most likely diagnosis?

A. caecal volvulus

B. sigmoid volvulus

C. paralytic ileus

D. large bowel obstruction due to distal malignancy

E. small bowel malrotation and volvulus

A

B. sigmoid volvulus

Sigmoid volvulus usually occurs when the sigmoid loop twists around its mesenteric axis, creating a closed loop obstruction.

Typical features are of an inverted U-shaped loop converging on the left side of the pelvis.

The bowel loop is usually markedly distended, appears ahaustral, and may overlap the lower border of the liver (liver overlap sign) or the haustrated dilated descending colon (left flank overlaps sign).

The apex of the volvulus usually lies under the left hemidiaphragm with its apex above the level of T10.

Caecal volvulus occurs when the caecum is on a mesentery, and involves the caecum either twisting and inverting so the caecal pole lies in the left upper quadrant, or twisting in an axial plane so that the caecum remains right sided or central.

Appearances are of a large, gas-distended viscus, usually with haustral markings, and occasionally the gas-filled appendix may be identified.

How well did you know this?
1
Not at all
2
3
4
5
Perfectly
16
Q
  1. A 34-year-old woman presents with bloody diarrhoea and abdominal pain. Which feature on barium enema favors a diagnosis of ulcerative colitis rather than Crohn’s disease?

A. thickened ileocecal valve

B. circumferential wall involvement

C. fistula formation

D. skip lesions

E. normal rectum

A

B. circumferential wall involvement

Ulcerative colitis and Crohn’s disease are idiopathic inflammatory diseases of the bowel.

Ulcerative colitis predominantly involves the mucosa and submucosa, and characteristically produces continuous, circumferential involvement of the colon.

Crohn’s disease produces transmural inflammation, may affect the entire gastrointestinal tract, and is characterized by eccentric and discontinuous involvement.

Typical features of ulcerative colitis include predominantly left-sided colonic involvement with rectosigmoid involvement in 95% of cases, a patulous ileocecal valve and shallow ulceration.

Typical features of Crohn’s disease include skip lesions (discontinuous disease), terminal ileal involvement with a thickened ileocaecal valve and fistula formation.

How well did you know this?
1
Not at all
2
3
4
5
Perfectly
17
Q
  1. A 23-year-old man presents with acute lower abdominal pain. An abdominal radiograph demonstrates a rounded, laminated calcific density projected over the right lower quadrant. What is the approximate likelihood of a diagnosis of acute appendicitis?

A. 10%

B. 30%

C. 50%

D. 70%

E. 90%

A

E. 90%

A laminated calcified appendicolith is seen in only 7–15% of patients with acute appendicitis. However, the presence of acute abdominal pain with an appendicolith on abdominal plain film indicates a 90% probability of acute appendicitis, and also indicates a high probability of gangrene/perforation. Other plain film signs of acute appendicitis include caecal wall thickening, small bowel obstruction and focal extraluminal gas collections.

How well did you know this?
1
Not at all
2
3
4
5
Perfectly
18
Q
  1. A 73-year-old man presents with lower abdominal pain and a change in bowel habit. A contrast enema demonstrates a stricture in the sigmoid colon. Which feature would favor a diagnosis of colorectal carcinoma rather than diverticulitis?

A. long (>10 cm) segment of involvement

B. mucosal ulceration

C. presence of a Colo vesical fistula

D. multiple diverticula in the sigmoid colon

E. smoothly tapered stricture margins

A

B. mucosal ulceration

The differentiation between complicated diverticular disease and a perforating colorectal carcinoma may be difficult.

Both may appear as a focal area of eccentric luminal narrowing on contrast enema.

A longer segment of involvement favors diverticulitis, as well as other inflammatory causes of colitis.

In addition, stricture margins in diverticulitis tend to be smoothly tapered rather than the abrupt narrowing of carcinoma.

Mucosal ulceration occurs in most cases of colorectal carcinoma, and is not a particular feature of diverticulitis.

Fistula formation may occur in both conditions, most commonly between the colon and the bladder, though this is more commonly seen in diverticulitis.

The presence of diverticula does not exclude the possibility of colorectal carcinoma, and the two conditions may coexist, as both are common in elderly people

How well did you know this?
1
Not at all
2
3
4
5
Perfectly
19
Q
  1. In patients with cystic fibrosis, which gastrointestinal pathology may occur as a result of high dose lipase supplementation?

A. rectal prolapse

B. fibrosing colonopathy

C. pneumatosis intestinalis

D. gastro-oesophageal reflux

E. meconium ileus equivalent syndrome

A

B. fibrosing colonopathy

Fibrosing colonopathy is a condition causing progressive submucosal fibrosis predominantly affecting the proximal colon. It was first described in 1994 in children with cystic fibrosis taking high-dose lipase supplementation to relieve the symptoms of exocrine pancreatic insufficiency. It causes structuring and longitudinal shortening of the right colon, and patients present with obstruction. Overall, the gastrointestinal tract is affected in 85–90% of patients with cystic fibrosis, and all of the above pathologies may occur, though only fibrosing colonopathy is associated with high-dose lipase supplementation.

How well did you know this?
1
Not at all
2
3
4
5
Perfectly
20
Q
  1. A 73-year-old woman presents with intermittent lower gastrointestinal bleeding and iron deficiency anaemia. She is clinically suspected to have angiodysplasia. What are the most likely findings on barium enema?

A. normal appearances

B. multiple small polyps in the colon

C. multiple shallow ulcers in the colon

D. multiple, serpiginous, filling defects in the colon

E. a focal, irregular, circumferential narrowing in the colon

A

A. normal appearances

In angiodysplasia, there is degenerative dilatation of the normal vessels in the submucosa of the bowel wall. It is associated with increasing age, and in about 20% of cases with aortic stenosis. It occurs most commonly in the right colon and presents with intermittent, low-grade bleeding. Barium enema shows no abnormality, as the lesion is submucosal, but increased tracer accumulation may be seen at the site of haemorrhageon99 mTc-labelled red cell scanning.

Angiography, if performed, may demonstrate a cluster of vessels along the antimesenteric border during the arterial phase and early opacification of the draining ileocolic vein.

How well did you know this?
1
Not at all
2
3
4
5
Perfectly
21
Q
  1. A 50-year-old woman undergoes CT colonography for a change in bowel habit, which demonstrates a well-defined, broad-based, 3 cm submucosal mass of density –40 HU in the ascending colon. It is noted to change shape between prone and supine images. What is the most likely diagnosis?

A. adenomatous polyp

B. endometriosis

C. primary pneumatosis coli

D. lipoma

E. enteric duplication cyst

A

D. lipoma

The colon is the commonest site for gastrointestinal lipomas. They typically appear as abroad-based mass, but may develop a short pedicle as a result of repeated peristaltic activity. They are soft lesions, and a change in shape or size may be noted on compression. At CT colonography, endoluminal features are non-specific, but the demonstration of fat attenuation on 2D images is diagnostic.

Primary pneumatosis coli usually appears as a cluster of air-filled cysts in the left colon. These may mimic polyps at endoscopy or on endoluminal CT colonography, but the demonstration of air attenuation on 2D images is again diagnostic.

Endometriosis and enteric duplication cyst are causes of submucosal lesions but are of soft-tissue and fluid density respectively.

Adenomatous polyps are mucosal lesions of soft-tissue density.

How well did you know this?
1
Not at all
2
3
4
5
Perfectly
22
Q

@# (Ped) 48) A 17-year-old girl presents with abdominal pain and rectal bleeding. She undergoes colonoscopy, which demonstrates multiple polypoid lesions in the colon. Which feature would favour a diagnosis of juvenile polyposis rather than familial adenomatous polyposis?

a. a total of 10 polyps in the colon

b. a histological diagnosis of tubulovillous polyps

c. involvement of the rectum

d. mucocutaneous pigmentation

e. a first-degree relative with multiple colonic polyps

A

a. a total of 10 polyps in the colon

In familial adenomatous polyposis (FAP), multiple (usually around 1000) tubular or tubulovillous adenomatous polyps are seen in the GI tract, predominantly in the colon.

Patients usually become symptomatic in the third to fourth decades and present with abdominal pain, weight loss and diarrhoea.

Juvenile polyposis (JP) is the commonest cause of colonic polyps in children, and usually presents with rectal bleeding. The polyps are hamartomatous and may occur throughout the GI tract. They are less numerous than in FAP, and the condition may be diagnosed with five or more polyps.

Both conditions are autosomal dominant, with 80% penetrance in FAP and variable penetrance in JP.

The rectosigmoid is involved in 80% of cases of JP, whereas the rectum is always involved in FAP. In both conditions, patients are at increased risk of associated adenocarcinoma, seen in 15% of patients by 35 years of age in JP, but in 100% of patients by 20 years after diagnosis in FAP.

Mucocutaneous pigmentation is a feature of Peutz–Jeghers syndrome.

How well did you know this?
1
Not at all
2
3
4
5
Perfectly
23
Q
  1. In the staging of rectal cancer by MRI, which sequence provides optimum visualization of the tumour?

A. T1W

B. contrast-enhanced T1W

C. T2W

D. FLAIR

E. proton density

A

C. T2W

MR is a highly accurate method of local staging of rectal cancer, with better assessment of locoregional nodal involvement than CT and clear depiction of the mesorecta fascia, allowing accurate prediction of whether the circumferential resection margin will be tumour free. T2W images provide optimal visualization of the tumour, which appears as an intermediate signal-intensity mass. Contrast-enhanced T1W images result in enhancement of the normal bowel wall as well as the tumour, which may lead to upstaging. FLAIR sequences are not routinely used for rectal cancer staging.

How well did you know this?
1
Not at all
2
3
4
5
Perfectly
24
Q
  1. A 67-year-old man presents with a sensation of incomplete evacuation and passage of thick mucus per rectum. Serum electrolytes demonstrate hypokalemia and hyponatremia. He undergoes barium enema, which demonstrates a broad-based, papillary, 2 cm lesion in the rectum with poor mucosal coating of barium. What is the most likely diagnosis?

A. lipoma

B. tubular adenoma

C. villous adenoma

D. colorectal carcinoma

E. solitary rectal ulcer syndrome

A

C. villous adenoma

Villous adenomas are a histological subtype of adenomatous polyps with predominantly villous elements, representing 10% of adenomatous polyps. Typical appearances are of abroad-based lesion often over 2 cm in diameter, with frond-like surface projections. They have a higher malignant potential than the other subtypes of adenomatous polyp (tubular and tubulovillous adenomas), which increases further with the size of the adenoma. Lesions under 5 cm have a 9% risk of malignant transformation to adenocarcinoma, whereas lesions over 10 cm have a 100%risk. Villous adenomas are associated with excretion of large amounts of thick mucus, which may result in diarrhoea and electrolyte depletion, as well as poor mucosal coating at barium enema.
Lipomas are typically seen as smooth, rounded, submucosal masses. Tubular adenomas are usually <10 mm and, like lipomas, are not associated with electrolyte depletion.
Solitary rectal ulcer syndrome may appear as polypoid lesions in the rectum and be associated with mucus secretion; however, it is normally associated with ulceration and is usually seen in young women.

How well did you know this?
1
Not at all
2
3
4
5
Perfectly
25
Q
  1. A 45-year-old woman with a previous history of treatment for advanced carcinoma of the cervix 8 years earlier presents with constipation and rectal bleeding. She undergoes CT of the abdomen and pelvis, which demonstrates a narrowed rectum with symmetrical wall thickening, perirectal inflammatory changes, thickening of the perirectal fascia and an increase in the AP diameter of the presacral space. What is the most likely diagnosis?

A. colorectal carcinoma

B. ulcerative colitis

C. radiation injury of the rectum

D. Hirschsprung’s disease

E. lymphoma

A

The answer is: C

Gastrointestinal complications following external radiotherapy are becoming more frequent as survival rates of patients with abdominal cancer improve, and they may present up to 15 years following irradiation. The colon and rectum are commonly affected following irradiation for pelvic and genitourinary tract malignancies. Chronic radiation colitis usually presents with strictures, whereas rectal injury manifests as a narrowed, thickened, poorly distensible rectum, with proliferation of the perirectal fat and thickening of the perirectal fascia. There is an increased incidence of colorectal carcinoma following pelvic irradiation, but a short irregular segment of narrowing would be more likely. Inflammatory bowel disease may result in circumferential rectal wall thickening with widening of the presacral space but is less likely given this history. Hirschsprung’s disease usually presents in early childhood. The rectum is an uncommon site for gastrointestinal lymphoma.

How well did you know this?
1
Not at all
2
3
4
5
Perfectly
26
Q

(Ped) 99) A 9-year-old boy with leukaemia and severe neutropenia presents with right lower quadrant abdominal pain and bloody diarrhoea. CT of the abdomen demonstrates circumferential thickening of the caecum with decreased bowel wall attenuation and pericolonic inflammatory changes. What is the most likely diagnosis?

a. appendicitis

b. typhlitis

c. leukaemic infiltration of the bowel

d. diverticulitis

e. Crohn’s disease

A

b. typhlitis

Typhlitis is acute inflammation of the caecum, appendix and occasionally terminal ileum, initially described in neutropenic children with leukaemia, but also seen with lymphoma, following immunosuppressive therapy and with clinical AIDS. Patients present with abdominal pain and diarrhoea, and may have a palpable, right lower quadrant mass. Characteristic findings are of circumferential caecal wall thickening, with oedematous bowel wall and inflammatory changes. Pericolonic fluid and intramural pneumatosis may be seen. Leukaemic deposits would be expected to cause more eccentric bowel wall thickening. Appendicitis may result in apical caecal wall thickening but would be accompanied by abnormal appendix. Crohn’s disease may produce a similar picture, but, in this clinical setting, typhlitis is the most likely diagnosis. Diverticulitis would be very unlikely in this age group.

How well did you know this?
1
Not at all
2
3
4
5
Perfectly
27
Q

In the staging of colorectal carcinoma, inferior mesenteric lymph nodes are considered distant metastasis rather than regional drainage for which tumour site?

(a) Ascending colon

(b) Transverse colon

(c) Descending colon

(d) Sigmoid colon

(e) Rectum

A

(a) Ascending colon

The ileocolic, right colic and middle colic lymph nodes are the regional drainage for the ascending colon (following arterial blood supply). All other nodal groups are considered distant metastases.

How well did you know this?
1
Not at all
2
3
4
5
Perfectly
28
Q

Which of the following tumours of the vermiform appendix is encountered most commonly?

(a) Adenocarcinoma

(b) Carcinoid

(c) lymphoma

(d) Mucinous adenocarcinoma

(e) Gastrointestinal Stromal Tumour

A

(b) Carcinoid

Seen in up to 1.4%of histology specimens and is usually incidental. Other tumours are encountered less commonly. The presence of adenocarcinoma may necessitate a formal right hemicolectomy to resect the draining lymph nodes.

How well did you know this?
1
Not at all
2
3
4
5
Perfectly
29
Q

Which of the following statements regarding the internal anal sphincter is true?

(a) It is made of striated muscle.

(b) Isolated injury is usually due to obstetric injury

(c) It appears hyperechoic on endoanal ultrasound

(d) it is the termination of the circular smooth muscle of the gastrointestinal tract

(e) The deep part fuses with the puborectalis sling and levator muscle.

A

(d) it is the termination of the circular smooth muscle of the gastrointestinal tract

The IAS comprises smooth muscle and is not under voluntary control. It appears hypoechoic on US. Obstetric injuries involve the external anal sphincter +/- IAS. Isolated injury is due to endo-anal trauma e.g. hemorrhoid surgery. Answers (a) and (e) pertain to the external anal sphincter, which is under voluntary control.

How well did you know this?
1
Not at all
2
3
4
5
Perfectly
30
Q

A 39-year-old man presents with epigastric pain, diarrhoea, PR bleeding, exhaustion, and fatigue. He is noted to have a swelling of the jaw. On examination, there are several calvarial lumps. CT head shows sebaceous cysts and bone lesions which are likely osteomas. OGD shows gastric hamartomas, colonoscopy shows multiple polyps throughout the colon.
What is the most likely diagnosis?

(a) Cowden disease

(b) Gardner’s syndrome

(c) Lynch syndrome

(d) Peutz-Jegher’s syndrome

(e) Turcot’s syndrome

A

(b) Gardner’s syndrome

5%Of colorectal carcinoma is genetic in origin; the most common inherited syndromes being familial adenomatous polyposis and hereditary non-polyposis colorectal cancer (Lynch syndrome; HNPCC).
Gardner’s syndrome is associated with polyposis in the colon (100%), duodenum (90%) and, rarely other bowel segments; there is an association with gastric hamartomas, osteomas in calvarium/ mandible and soft tissue tumours (30%).
Turcot syndrome is a rare condition which is also associated with CNS gliomas and medulloblastomas.
Peutz-Jegher’s syndrome consists of hamartomas throughout the Gl system with the rare potential for malignant transformation, and perioral pigmentation.
Cowden disease has hamartomas, gingival hyperplasia, oral papillomas, muco-cutaneous pigmentation and an increased risk of breast and thyroid malignancy.

How well did you know this?
1
Not at all
2
3
4
5
Perfectly
31
Q

A previously well 65.year.old man has a myocardial infarction. Six days later he develops acute abdominal pain and distension; he feels nauseated and vomits. An abdominal film demonstrates distended proximal colon to the splenic flexure and normal colon distally. A contrast enema is performed with no abnormality seen. What is the most likely diagnosis?

(a) Sigmoid volvulus

(b) Adhesions causing large bowel obstruction

(c) Caecal volvulus

(d) Toxic megacolon

(e) Acute colonic pseudo-obstruction

A

(e) Acute colonic pseudo-obstruction

Acute colonic pseudo-obstruction (Ogilvie syndrome) is a rare condition that may be seen most commonly after trauma (including surgery), infection or with cardiac disease. The features mimic obstruction and complications include perforation. Gastrograffin enema may be therapeutic; air should never be insufflated in to the colon in these cases.

How well did you know this?
1
Not at all
2
3
4
5
Perfectly
32
Q

How many types of caecal volvulus are described?

(a) 1

(b) 2

(c) 3

(d) 4

(e) 5

A

(c) 3

In the axial torsion type, the caecum twists around its long axis and remains in the right lower quadrant. In the loop type, the caecum twists around its long axis and also inverts. In the caecal bascule type, the caecum folds anteromedial to the ascending colon with no torsion. In the latter two scenarios, the caecum is found in the upper abdomen.

How well did you know this?
1
Not at all
2
3
4
5
Perfectly
33
Q

A 37-year-old lady with longstanding constipation is referred for a defecating proctography. The only abnormality seen on resulting images an 8 cm anterior bulge of the anterior margin of the rectum with incomplete evacuation. What is the diagnosis?

(a) Rectal prolapse

(b) Rectocele

(c) tnismus

(d) Cystocele

(e) Enterocoele

A

(b) Rectocele

An anterior bulge of up to 3 or 4 cm may be normal in many cases, but beyond this, and with incomplete evacuation, the diagnosis of a rectocoelemay be made. Cystocoele and enterocoele are the abnormal descent of bladder and small bowel respectively. Prolapse is the abnormal descent of the rectum. Anismus is a functional abnormality leading to poor co-cordination Of the pelvic floor muscles.

How well did you know this?
1
Not at all
2
3
4
5
Perfectly
34
Q

A patient with Crohn’s disease is referred for a small bowel enema. Which of the following features would you not expect to see?

(a) Aphthoid ulcers

(b) Kinked bowel segments

(c) Sacculation

(d) Increased number of folds in the ileum

(e) Loop separation

A

(d) Increased number of folds in the ileum

Jejunistion of the ileum is a feature of coeliac disease. In addition to the other features, transmural ulceration, fistulae, stenosis and fold thickening may be seen. Crohn’s disease may affect any part Of the GI tract, from mouth to anus.

How well did you know this?
1
Not at all
2
3
4
5
Perfectly
35
Q

A 13-year-old boy is undergoing his final course of chemotherapy for lymphoma. He presents to the clinic with a fever and acute right iliac fossa pain. Contrast-enhanced CT shows enhancement and circumferential thickening of the caecal wall, appendix and terminal ileum surrounding fat stranding and free fluid.
What is the likely diagnosis?

(a) Appendicitis

(b) Lymphomatous infiltration

(c) Mycobacteriurr7 avium-intracellulare

(d) Sclerosing mesenteritis

(e) Typhlitis

A

(e) Typhlitis

Typhlitis (neutropenic colitis) represents acute inflammation of the caecum, appendix and occasionally terminal ileum. The aetiology is unknown, but profound neutropaenia appears to be universal. Mucosal injury from cytotoxic drugs during chemotherapy is thought to play an important role; infection may be involved (e.g.CMV). Lymphomatous deposits would be expected to produce a more eccentric thickening.

How well did you know this?
1
Not at all
2
3
4
5
Perfectly
36
Q

A 40-year-old man presents with acute left iliac fossa pain. He is afebrile and the WCC is normal. CT demonstrates a 2.5 cm ovoid lesion with surrounding fat stranding arising from the anterior wall of the sigmoid colon. The lesion enhances peripherally and has a central attenuation of –50 HU. There is no associated thickening of the colonic wall. What is the most likely diagnosis?

(a) Diverticulitis

(b) Epiploic appendagitis

(c) Mesenteric panniculitis

(d) Omental infarct

(e) Sclerosing mesenteritis

A

(b) Epiploic appendagitis

Epiploic appendages are peritoneal fat outpouchings that arise from the serosal surface of the colon, attached by vascular stalks (supplied by 1-2 small end-arteries and a draining vein). They contain adipose tissue and vessels, and measure up to 5 cm (typically 1-2 cm). Epiploic appendagitis is inflammation secondary to torsion or venous occlusion. The most common location is anterior to the sigmoid colon, there is surrounding fat stranding, and a low-density (fat) center is seen; associated colon thickening is rare.
Omental infarction can appear similar, but lacks the hyperdense ring enhancement and is more typically seen as an oval soft-tissue mass in the right lower quadrant, deep to the anterior abdominal muscles.

How well did you know this?
1
Not at all
2
3
4
5
Perfectly
37
Q

Which of the following polyposis syndromes is not associated with malignancy?

(a) Gardner’s syndrome

(b) Cronkhite Canada syndrome

(c) Turcot’s syndrome

(d) Cowden’s disease

(e) Peutz-Jegher’s disease

A

(b) Cronkhite Canada syndrome

Cronkhite Canada syndrome is a rare non-familial condition with multiple hamartomatous polyps resulting in a protein-losing enteropathy. It is seen most commonly in Japan and the underlying cause remains unclear.

How well did you know this?
1
Not at all
2
3
4
5
Perfectly
38
Q

What is the most common site for actinomycosis infection within the abdomen?

(a) Appendix

(b) Liver

(c) Spleen

(d) Terminal ilium

(e) Jejunum

A

(a) Appendix

Caused by Actinomycoces israelii, this is a rare condition which clinically mimics an appendix abscess. Sinus tracks, fistulae and a mass may all be seen at imaging. Risk factors include previous abdominal surgery, GI perforation, diabetes mellitus and steroid therapy.

How well did you know this?
1
Not at all
2
3
4
5
Perfectly
39
Q

A patient undergoes local staging of rectal carcinoma by MRI. The report concludes that there is “a circumferential tumour extending beyond the muscularis layer which is 1 mm away from the meso-rectal fascia.” How might this also be expressed?

(a) T3 disease threatening the circumferential resection margin

(b) T3 disease involving the circumferential resection margin

(c) T4 disease threatening the circumferential resection margin

(d) T4 disease involving the circumferential resection margin

(e) None of the above

A

(a) T3 disease threatening the circumferential resection margin

Extension through the muscularis layer in to the mesorectal fat without invasion of other structures is T3 disease. Surgeons operating on rectal cancer perform a total mesorectal excision (TME) and remove the mesorectumen bloc. At least 3 clearance is required on MRI to confidently predict that the circumferential margin will be negative following TME. Neo-adjuvant chemo/radiotherapy is usually given to patients in this instance.

How well did you know this?
1
Not at all
2
3
4
5
Perfectly
40
Q

A 48-year-old man undergoes a CT colonography study which identifies a single polyp measuring 11 mm in diameter. What is the likelihood that this polyp will contain a focus of carcinoma?

(a) 0.1 %

(b) 0.8 %

(c) 2.6 %

(d) 6.6 %

(e) 12.3 %

A

(c) 2.6 %

The likelihood of there being advanced neoplasia (high grade dysplasia or > 25% villous adenoma) or malignancy in a polyp is, respectively, 1.7% and <0.1% for a polyp <5mm, 6.6% and 0.2% for a polyp measuring 6-9 mm, and 30.6% and 2.6% for a polyp measuring 1 cm or more.

How well did you know this?
1
Not at all
2
3
4
5
Perfectly
41
Q

In patients with AIDS, which portion of the GI tract is most commonly affected by cytomegalovirus infection?

(a) Oesophagus

(b) Stomach

(c) Jejunum

(d) Ileum

(e) Colon

A

(e) Colon

CMV is one of the most common causes of enteric disease in AIDS patients, usually affecting those with a CD4 count < 100. The colon is the most commonly affected site, followed by the small bowel, oesophagus and stomach.

How well did you know this?
1
Not at all
2
3
4
5
Perfectly
42
Q

A 67-year-old lady is referred for staging of anal carcinoma. On MRI, the tumour measures 5.5 cm in diameter and invades the internal sphincter, but not the external sphincter. What is the T-stage of the tumour?

(a) T1

(b) T2

(c) T3

(d) T4

(e) T5

A

(c) T3

Lesions <2 cm are TI
2-5 cm are T2
> 5 cm are T3.
T4 lesions are those invading adjacent organs.
There is no T 5 category.
Depth of invasion is irrelevant with the exception of T4 disease.

How well did you know this?
1
Not at all
2
3
4
5
Perfectly
43
Q

Which of the following imaging features is more suggestive of a diagnosis Of Ulcerative colitis rather than Crohn’s colitis?

(a) Multiple anal fistulae

(b) Aphthoid ulceration

(c) Enlarged lymph nodes

(d) Entero-enteric fistulae

(e) Granularity

A

(e) Granularity

The granular appearance on barium enema is typical. UC is typically contiguous from the rectum and appears symmetrical. In chronic dis, fibro-fatty proliferation is seen in mesorectum, there is submucosal fat deposition.

How well did you know this?
1
Not at all
2
3
4
5
Perfectly
44
Q

What is the most common site of colonic lipomas?

(a) Caecum

(b) Transverse colon

(c) Descending colon I

(d) Sigmoid colon

(e) Rectum

A

(a) Caecum

Lipomas are relatively common sub-mucosal lesions in the GI tract, seen in 4.4% of patients at autopsy, and may be solitary or multiple. the colon, 45% of these are seen in the caecum.

How well did you know this?
1
Not at all
2
3
4
5
Perfectly
45
Q

A 39-year-old man is referred for MRI examination anal fistula. This demonstrates a fluid track extending from the perineum on the right, running medial to the external anal sphincter throughout its course and entering the at the 6 o’clock position. How should you report this?

(a) Horseshoe fistula

(b) Extrasphincteric fistula

(c) Transphincteric fistula

(d) Suprasphincteric fistula

(e) Intersphincteric fistula

A

(e) Intersphincteric fistula

There are 4 types of fistula (b) - (e), with the intersphincteric type the only one that does not pass through the external sphincter (hence it is always medial to it). If a fistula extends around both sides of the anal canal, this is termed a horseshoe extension.

How well did you know this?
1
Not at all
2
3
4
5
Perfectly
46
Q

A pregnant lady presents severe right iliac fossa pain. The surgical team request an MRI to evaluate the appendix. What imaging sequence is best for depicting the appendix?

(a) T1

(b) T2

(c) STIR

(d) FLAIR

(e) DWI

A

(b) T2

T2 weighted images best depict the appendix, but a STIR sequence is sensitive for identifying an inflamed and oedematous appendix. MRI has a sensitivity, specificity and accuracy of over 90% for the diagnosis of acute appendicitis.

How well did you know this?
1
Not at all
2
3
4
5
Perfectly
47
Q

A 78-year-old man with long history of constipation presents severe acute abdominal pain. A supine abdominal radiograph demonstrates a dilated loop of bowel extending from the pelvis and overlying the liver, reaching to the level of the D9 vertebra. The descending colon is markedly distended. What is the diagnosis?

(a) Giant diverticulum

(b) Megacolon

(c) Pseudo-obstruction

(d) Diverticulitis

(e) Sigmoid volvulus

A

(e) Sigmoid volvulus

These are the classical radiological features of sigmoid volvulus on the plain abdominal film. The sigmoid is prone to twisting as it has a short mesentery and when it does so, it rotates anti-clockwise and may become ischemic. It is seen more commonly in the elderly with constipation, in those with high fibre diets, and in children. Decompression alone will lead to representation in 50% within 2 years.

How well did you know this?
1
Not at all
2
3
4
5
Perfectly
48
Q

With regard to pseudomembranous colitis, which of the following is not true?

(a) Ascites is seen in 1/3 of cases

(b) The organism is usually confined to the mucosa

(c) It is associated with the use of proton pump inhibitors

(d) Skip lesions indicate co-infection or underlying Crohn’s disease

(e) It causes megacolon more frequently than ulcerative colitis

A

(d) Skip lesions indicate co-infection or underlying Crohn’s disease

Clostridium difficile, the organism which causes pseudomembranous colitis, acts through the release of endotoxins which produce diarrhoea and abdominal pain. The use of PPIs increases the survival of vegetative matter in the stomach and is strongly associated with this condition. The colon may be affected from the rectum to the caecum in a variety of patterns.

How well did you know this?
1
Not at all
2
3
4
5
Perfectly
49
Q

Which of the following extra-intestinal manifestations of inflammatory bowel disease is seen more commonly with ulcerative colitis rather than Crohn’s disease?

(a) Iritis

(b) Gallstones

(c) Sacroiliitis

(d) Nephrolithiasis

(e) Erythernanodosurn

A

(c) Sacroiliitis

Crohn’s disease is more commonly associated with a peripheral, migratory, non-deforming seronegative arthropathy.

How well did you know this?
1
Not at all
2
3
4
5
Perfectly
50
Q

With regard to diverticular disease, which of the following statements is true?

(a) The rectum is involved in approximately 5% cases

(b) Localised right colonic disease is seen in 20% cases

(c) Giant diverticula most commonly arise from the caecurn

(d) NSAIDs increase the risk of perforation

(e) Hemorrhage is most commonly due to sigmoid disease

A

(d) NSAIDs increase the risk of perforation

Rectal involvement is rare, with localised right colonic disease seen in around 5% of cases whilst sigmoid disease accounts for 80%. Giant diverticulae most commonly arise in the sigmoid colon, but bleeding is more common from the right colon.
Glucocorticoids also increase the risk of perforation.

How well did you know this?
1
Not at all
2
3
4
5
Perfectly
51
Q
  1. A 60-year-old female has a plain abdominal film which shows a grossly distended segment of bowel. Which one of the following features makes a diagnosis of caecal volvulus more likely than sigmoid volvulus?

a. Pelvic overlap sign

b. Apex lying above the level of T10

c. Liver overlap sign

d. Coffee bean sign

e. Presence of haustral markings

A
  1. e. Presence of haustral markings

Sigmoid and caecal volvulus can sometimes be difficult to differentiate on plain abdominal film. With caecal volvulus the haustral markings are typically present, whereas these are usually absent in sigmoid volvulus. The pelvic overlap, liver overlap and coffee bean signs are typical of sigmoid volvulus. In sigmoid volvulus the apex lies high in the abdomen underneath the left hemi-diaphragm, typically above the level of T10.

How well did you know this?
1
Not at all
2
3
4
5
Perfectly
52
Q
  1. A 40-year-old man is admitted to the surgical ward with acute abdominal pain and subsequently a CT abdomen and pelvis is requested. The findings include a 3 cm oval mass with central fat density adjacent to the sigmoid colon and with associated fat stranding. Which one of the following is the most likely diagnosis?

a. Diverticulitis

b. Epiploic appendagitis

c. Mesenteric lymphadenitis

d. Meckel’s diverticulitis

e. Infected enteric duplication cyst

A
  1. b. Epiploic appendagitis

Epiploic appendagitis is inflammation of one of the epiploic appendages of the colon, with the sigmoid being the commonest site. It typically presents with acute abdominal pain and is an important radiological diagnosis as it can often mimic appendicitis, and management is conservative. The diagnosis is usually made on CT with the features described in the question. Ultrasound is rarely used for diagnosis, and features include a non-compressible hyperechoic mass with hypoechoic margins.

How well did you know this?
1
Not at all
2
3
4
5
Perfectly
53
Q
  1. A seven-year-old boy on chemotherapy for acute leukemia develops severe right iliac fossa pain and diarrhoea. CT shows ascending colon and caecal wall thickening, with inflammation extending to involve the appendix and terminal ileum and fat stranding in the adjacent mesentery. The most likely diagnosis is:

a. Typhlitis

b. Crohn’s disease

c. Acute appendicitis

d. Necrotizing enterocolitis

e. Acute leukemic infiltration

A
  1. a. Typhlitis

Typhlitis, or neutropenic enterocolitis, is acute inflammation of the caecum, ascending colon, terminal ileum or appendix. It is typically described in children with neutropenia secondary to lymphoma, leukemia and immunosuppression. Concentric, often marked, bowel wall thickening with pericolic inflammatory changes is typical, and such changes in a young immunosuppressed child should raise suspicion of typhlitis as a cause. Perforation is a risk factor and therefore contrast examinations are usually avoided.

How well did you know this?
1
Not at all
2
3
4
5
Perfectly
54
Q
  1. A 71-year-old man is referred to CT for unexplained abdominal distension. Low attenuation intraperitoneal collections with enhancing septae are demonstrated. There is scalloping of the liver border and omental thickening. Which one of the following is most likely to be the underlying cause?

a. Carcinoid tumour of the appendix

b. Cystadenocarcinoma of the appendix

c. Melanosis coli

d. Mastocytosis

e. Retroperitoneal fibrosis

A
  1. b. Cystadenocarcinoma of the appendix

The CT findings described are consistent with pseudomyxoma peritonei. This describes abdominal distension secondary to the accumulation of large quantities of gelatinous ascites. It is most commonly caused by cystadenocarcinoma of the appendix in males and cystadenocarcinoma of the ovary in females. Surgical debulking and intraperitoneal chemotherapy may be offered as a treatment. Bowel obstruction is a frequent complication that may necessitate surgery.

How well did you know this?
1
Not at all
2
3
4
5
Perfectly
55
Q
  1. A 78-year-old previously well female is admitted with acute abdominal pain and diarrhoea. Contrast-enhanced CT of the abdomen and pelvis shows thickening of a 13 cm segment of proximal descending colon and mucosal hyperenhancement. The rest of the colon is normal, and the small bowel is unaffected. There is a small amount of free fluid in the pelvis. Which one of the following diagnoses is most likely?

a. Crohn’s colitis

b. Ulcerative colitis

c. Ischaemic colitis

d. Infectious colitis

e. Pseudomembranous colitis

A
  1. c. Ischaemic colitis

Crohn’s colitis is relatively unlikely due to lack of prior history or small bowel involvement and age of the patient. Ulcerative colitis and pseudomembranous colitis are both unlikely as the rectum is usually involved in these two conditions. Infectious colitis does not normally affect the left-sided colon only, regardless of the underlying pathogen. Ischaemic colitis is the most likely diagnosis of those listed. It typically affects a segment of bowel, with the majority of cases having left-sided colonic involvement.

How well did you know this?
1
Not at all
2
3
4
5
Perfectly
56
Q
  1. A 25-year-old male presents with abdominal cramps and pain with rectal bleeding. Colonoscopy is normal. CT enteroclysis is performed as part of the investigation, which reveals multiple sessile polyps throughout the jejunum and ileum. Subsequent biopsies reveal these polyps to be hamartomas. Which one of the following syndromes is he most likely to be diagnosed with?

a. Peutz–Jegher’s

b. Cowden’s

c. Turcot’s

d. Familial polyposis

e. Gardner’s

A
  1. a. Peutz–Jegher’s

Peutz–Jegher’s syndrome is most consistent with these findings. It is an autosomal dominant syndrome but often arises as a spontaneous mutation. Hamartomas are found throughout the gastro-intestinal tract, with the exception of the oesophagus. The polyps have almost no malignant potential, but life expectancy is decreased due to associated cancers arising in the stomach, duodenum, colon and ovary. Gardner’s syndrome and familial polyposis are both associated with small bowel adenomas in approximately 5% of cases. Cowden’s syndrome does involve hamartomatous polyps, but these are typically rectosigmoid, and small bowel involvement is not a feature. Small bowel polyps are not a feature of Turcot’s syndrome.

How well did you know this?
1
Not at all
2
3
4
5
Perfectly
57
Q
  1. A 17-year-old female undergoes screening colonoscopy and is found to have multiple adenomatous polyps throughout the colon. OGD and biopsy reveal multiple hamartomas of the stomach and duodenum. She subsequently has investigation for a painful jaw that reveals a 1 cm round, discrete, dense lesion in the mandible. Which one of the following syndromes is the most likely underlying diagnosis?

a. Lynch syndrome

b. Cronkhite–Canada syndrome

c. Familial adenomatous polyposis

d. Gardner’s syndrome

e. Peutz–Jegher syndrome

A
  1. d. Gardner’s syndrome

Gardner’s syndrome is an autosomal dominant condition with colonic polyps present in all patients. Small bowel, duodenal and stomach polyps are also a feature. Extra-intestinal features include osteomas of membranous bone (typically the mandible as described in the question), other soft-tissue tumours and periampullary carcinomas. Osteomas are not a feature of the other conditions. Cronkhite–Canada syndrome and Peutz–Jegher syndrome are associated with multiple hamartomatous polyps of the colon and stomach. Cronkhite– Canada syndrome is a sporadic non-familial disorder. Lynch syndrome, or hereditary nonpolyposis colorectal carcinoma (HNPCC), is associated with increased risk of colorectal adenomas and other malignancies such as endometrial and other gastro-intestinal tract malignancies.

How well did you know this?
1
Not at all
2
3
4
5
Perfectly
58
Q
  1. A 35-year-old male with known ulcerative colitis presents to A&E with severe abdominal pain, pyrexia and diarrhoea. There is no peritonism. Toxic megacolon is suspected clinically. Which one of the following is the most appropriate as first line imaging?

a. CT

b. Plain abdominal film

c. Double contrast barium enema

d. Single contrast water-soluble enema

e. Targeted bowel ultrasound

A
  1. b. Plain abdominal film

Toxic megacolon is a complication of ulcerative colitis, Crohn’s and other forms of acute colitis. It has a poor prognosis with up to 20% mortality. Plain abdominal radiography should be the first line investigation for suspected toxic megacolon, and can be repeated 24 or 48 hourly if necessary. It can often confirm the diagnosis without the need for CT, which is especially useful when considering radiation dose issues in this group of young patients. Typical features on plain film include transverse colon dilatation >5.5 cm, loss of normal haustral folds, thumbprinting of the colon and the presence of mucosal islands (pseudopolyps). CT better demonstrates potential complications of toxic megacolon such as perforation of the bowel.

59
Q
  1. As part of an investigation for altered bowel habit, a 32-year-old female has a double contrast barium enema performed. Findings include deep and superficial aphthous ulceration from the caecum proximally to the sigmoid colon and the presence of pseudodiverticula. Which one of the following is most likely?

a. Crohn’s disease

b. Ulcerative colitis

c. Tuberculosis

d. Yersinia

e. Lymphoma

A
  1. a. Crohn’s disease

These features are highly suggestive of Crohn’s disease. Signs on double contrast barium enema that favor a diagnosis of Crohn’s disease over ulcerative colitis include apthoid ulcers, deep ulcers, discontinuous ulceration, rectal sparing, pseudodiverticulae, fistulae and abscess formation. Ulcerative colitis can be suggested by rectal involvement, continuous pathology with no skip lesions and the presence of mucosal granularity. However, these features may also be present in Crohn’s and are not specific for ulcerative colitis. Although tuberculosis is a mimic, colonic involvement in this pattern is uncommon compared with Crohn’s disease.

60
Q
  1. An 81-year-old man is investigated for anaemia of unknown cause. He has a barium enema as an outpatient that is reported as normal, but is subsequently admitted with a large gastro-intestinal bleed. Initial upper gastro-intestinal endoscopy is normal. He is hemodynamically unstable and therefore has a mesenteric angiogram, which shows early opacification and slow emptying of the ileocolic vein. Which one of the following diagnoses is most likely?

a. Angiodysplasia

b. Diverticulosis

c. Meckel’s diverticulum

d. Adenomatous polyp

e. Radiation enteritis

A
  1. a. Angiodysplasia

The typical angiographic feature of a Meckel’s diverticulum is presence of the vitelline artery. Meckel’s diverticulum also typically (although not exclusively) presents in younger patients. With diverticulosis, radiation enteritis and polyps, one might expect an abnormal barium enema, and in addition these angiographic features are not typical. Angiodysplasia is the second most common cause of gastro-intestinal bleed in the elderly population after diverticular disease. It is due to dilatation of the submucosal vessels and occurs most commonly on the right side of the colon. Angiographic features include visualisation of a cluster of vessels on the antimesenteric border, early filling of the ileocolic vein in the arterial phase and delayed emptying of the same vein.

61
Q
  1. A 37 year old female has a pelvic MRI for investigation of rectal pain and bleeding, following a normal flexible sigmoidoscopy. This shows a thin-walled dumbbell-shaped 7 cm multilocular cyst. It is in contact anteriorly to the rectum and posteriorly to the presacral fascia, but contained within the mesorectal fascia. Although the rectum is distorted by the mass, the rectum and sigmoid are normal. What is the most likely diagnosis?

a. Rectal duplication cyst

b. Anterior sacral meningocoele

c. Mucinous rectal carcinoma

d. Presacral dermoid

e. Tailgut cyst

A
  1. e. Tailgut cyst

Tailgut cysts or cystic hamartomas are presacral, multilocular, mucous-secreting cysts found distal to the normal embryonic termination of the hindgut. Small cysts may be asymptomatic, but larger cysts may present with rectal pain or bleeding, constipation, anal fistulae or recurrent rectal abscesses. A long and tail-like coccyx is often associated, and can help distinguish between many other cystic lesions in this region. Malignant transformation is a complication, most commonly adenocarcinoma. Duplication cysts are most often unilocular unless complicated by haemorrhage or infection, a meningocele is likely to arise from the sacral foramina, piercing the presacral fascia, and dermoid tumour would be expected to contain fat or layering of contents as seen in dermoid tumours in otherlocations.

62
Q
  1. A 76 year old male on ITU has a CT abdomen and pelvis for the investigation of abdominal pain, pyrexia and diarrhoea. The CT reveals 12mm diffuse large bowel wall thickening with intense mucosal enhancement and low attenuation of the submucosa involving the entire colon including the rectum, and a small volume of ascites. Which one of the following diagnoses is the most likely to explain the above findings?

a. Crohn’s colitis

b. Pseudomembranous colitis

c. Ischaemic colitis

d. Yersinia

e. Giardiasis

A
  1. b. Pseudomembranous colitis

Pseudomembranous colitis results from overgrowth of Clostridium difficile most commonly due to broad spectrumantibiotic use in the hospital population.Ascites is often present in severe cases and wall thickening >10mm is highly suggestive of this diagnosis. A layered pattern of enhancement is often present in severe cases with oedema in the submucosa producing low attenuation in the wall, deep to the enhancing mucosa. The accordion sign is caused by marked submucosal oedema producing thickening of the colonic haustra. The rectum is involved in the majority of cases but any location within the large bowel may be involved.

63
Q
  1. An ultrasound of the abdomen is performed on a 21 year old female presenting to A&E with acute right iliac fossa pain, pyrexia, tenderness and guarding. Which one of the following findings would suggest perforation of the appendix?

a. Appendix diameter of 8mm

b. Appendix wall thickness of 4mm

c. Decreased resistance of arterial waveform

d. Loss of visualisation of hyperechoic submucosa

e. Increased echogenicity of surrounding fat

A
  1. d. Loss of visualisation of hyperechoic submucosa

The use of ultrasound for the diagnosis of acute appendicitis is particularly useful in children and women of child-bearing age. Findings indicating acute appendicitis include a tubular non-compressible blind-ending structure with diameter >6 mm and wall thickness >2 mm, although these signs do not necessarily indicate perforation. Features suggesting perforation include a fluid collection adjacent to the appendix, gas bubbles near the appendix and loss of visualisation of the submucosal layer.

64
Q
  1. A 52 year old male with a metal heart valve has a transrectal ultrasound performed to stage rectal carcinoma as MRI is contraindicated. A 3 cm hypoechoic mass is identified from three to seven o’clock in the lower rectum. It extends through an inner hypoechoic layer and into the outer hypoechoic layer, but the outermost hyperechoic layer is intact and unaffected. What is the correct T staging (TNM system) based on these observations?

a. T0

b. T1

c. T2

d. T3

e. T4

A
  1. c. T2

The layers of the rectum are well demonstrated at transrectal ultrasound. The innermost hyperechoic layer represents the balloon-mucosa interface, the middle hyperechoic layer represents the submucosa and the outermost hyperechoic layer represents the serosa. The tumour described in the question extends through the submucosa into the muscularispropria (outer hypoechoic layer) but does not involve the serosa. T1 disease is limited to the submucosa, T2 is limited to the muscularispropria, T3 extends through the serosa and T4 represents invasion of adjacent organs. The correct staging for tumour described in the question is therefore T2.

65
Q

@#e QUESTION 5
A 19-year-old female student presents with acute abdominal pain, elevated CRP and a low-grade temperature. On clinical examination, there is tenderness to light palpation in the right iliac fossa and the patient is febrile. A graded compression ultrasound examination is performed. Which one of the following statements is true?

A A transverse appendiceal diameter of 5 mm is diagnostic of acute appendicitis.

B The finding of a pelvic fluid collection makes a diagnosis of acute appendicitis unlikely.

C The presence of hyperechoic fat in the right iliac fossa makes a diagnosis of acute appendicitis unlikely.

D The sensitivity of graded compression ultrasound in suspected acute appendicitis is 75—90%.

E The specificity of graded compression ultrasound in suspected acute appendicitis is 35—50%.

A

D The sensitivity of graded compression ultrasound in suspected acute appendicitis is 75—90%.

Graded compression ultrasound of the appendix can avoid unnecessary surgery and ionising radiation particularly relevant for children and women of childbearing age. The finding of a non-compressible appendix with transverse diameter of 6 mm or greater is highly suggestive of acute appendicitis (specificity 86-100%). Other ultrasound findings include hyperechoic fat in the right iliac fossa, peri-appendiceal fluid or a pelvic fluid collection (appendiceal abscess)

66
Q

@#e QUESTION 6
A 42-year-old man presents to the Emergency Department with a 7-day history of severe bloody diarrhoea and abdominal pain. He has previously been fit and well with no significant medical history. On examination, the patient is dehydrated with generalized abdominal tenderness but no clinical evidence of peritonism. An abdominal radiograph is performed. Which radiographic finding would be most suggestive of a toxic megacolon?

A Caecum measuring 4.5 cm in diameter

B Multiple mucosal islands in a dilated transverse colon

C Pseudodiverticulae in the descending colon

D Thickened haustrae throughout the entire colon

E ‘Thumbprinting’ of the transverse and descending colon

A

B Multiple mucosal islands in a dilated transverse colon

The presence of severe ulceration leading to mucosal islands is a major sign of toxic megacolon (the other key finding is colonic dilatation > 5 cm).

67
Q

A 74-year-old man presents with an 8-week history of altered bowel habit and rectal bleeding. A flexible sigmoidoscopy demonstrates a malignant stricture in the rectum and biopsies confirm rectal adenocarcinoma. An MRI is performed and shows an annular neoplasm at 12 cm. The mass invades 4 mm beyond the rectal wall into the perirectal fat and infiltrates the peritoneal reflection anteriorly. There is a small volume of free peritoneal fluid. What is the radiological T stage?

A TX

B T1

C T2

D T3

E T4

A

E T4

Evidence of peritoneal invasion indicates stage T4 rectal cancer.

68
Q

A male patient is referred to the on-call surgical team with a 3-day history of generalised abdominal pain and vomiting. The patient has not opened his bowels for 2 days. Examination reveals a distended abdomen with increased bowel sounds. An abdominal radiograph is performed and demonstrates a large dilated loop of large bowel with several loops of dilated small bowel centrally. Which other feature would make a diagnosis of caecal volvulus more likely than that of sigmoid volvulus?

A Haustrae are visible in the gas-filled viscus.

B The apex of the viscus lies in the left upper quadrant.

C The patient is 75 years old.

D The patient is in long-term institutional care.

E The viscus rises above the level of the Tl0 vertebral body.

A

A Haustrae are visible in the gas-filled viscus.

The presence of haustrations in a dilated viscus and gas in the appendix are key to the diagnosis of caecal volvulus.

69
Q

@#e QUESTION 24
A 23-year-old woman complains of episodes of diarrhoea and rectal bleeding. Her father died of colorectal cancer aged 39. A double contrast barium enema is performed and demonstrates more than one hundred small polyps, measuring up to 5 mm in size, throughout the colon. An upper GI endoscopy demonstrates multiple polypoid lesions in the stomach and duodenum. What is the most likely diagnosis?

A Carcinoid syndrome

B Familial adenomatous polyposis

C Hereditary non-polyposis colorectal cancer

D Juvenile polyposis

E Peutz-Jeghers syndrome

A

B Familial adenomatous polyposis

Autosomal dominant condition with multiple colonic adenomas and 100%risk of colorectal carcinoma 20 years after diagnosis. Associated with hamartomas in stomach, gastric & duod adenomas & periampullary carcinoma.

70
Q

QUESTION 29
A 35-year-old woman is referred to the Radiology Department following the birth of her first child. The baby was delivered 8 days post-term and was a vaginal delivery following a prolonged labour and episiotomy. Two months later, the patient continues to experience faecal incontinence and an anal sphincter tear is suspected. Which investigation would be most useful to demonstrate anal sphincter damage?

A Barium evacuation proctogram

B CT colonography

C CT with rectal contrast media

D Endoanal ultrasound

E MRI of the pelvis with a body coil

A

D Endoanal ultrasound

High-frequency endosonography allows an accurate assessment of the four layers of the anal wall; superficial and deep mucosa, submucosa and muscularis propria.

71
Q

QUESTION 30
A 33-year-old woman presents to her GP with a one year history of intermittent rectal bleeding. She experiences regular episodes of fresh blood per rectum with associated lower abdominal pain, lasting several days at a time. A flexible sigmoidoscopy is normal. A double contrast barium enema is performed and demonstrates an irregular appearance of the anterior wall of the sigmoid colon with mild extrinsic mass effect. What is the most likely diagnosis?

A Carcinoma of the sigmoid colon

B Endometriosis

C Pelvic lipomatosis

D Radiation enteritis

E Solitary rectal ulcer syndrome

A

B Endometriosis

Involvement of the GI- tract is not uncommon in endometriosis and the sigmoid colon and pelvic small bowel loops are typical sites of involvement.

72
Q

QUESTION 44
A 47-year-old man is knocked off his motorcycle and brought to the Emergency Department. On examination, he is haemodynamically stable but has left upper quadrant tenderness. A contrast-enhanced CT of the abdomen is performed and shows no evidence of visceral injury. The reporting radiologist notices a solitary, well-defined lesion in the large bowel that is of lower attenuation than the surrounding colonic wall. Which single additional finding would be most consistent with a colonic lipoma?

A There is a mean density of —10 HU on CT.

B Mucosal ulceration is seen on colonoscopy.

C On ultrasound, the lesion changes shape when compressed.

D The lesion lies in the sigmoid colon.

E There is a one-month history of rectal bleeding and weight loss.

A

C On ultrasound, the lesion changes shape when compressed.

Known as the ‘squeeze sign’.

73
Q

QUESTION 51
A 66-year-old man undergoes screening for colorectal cancer and is found to have two positive stool samples for faecal occult blood. The patient is asymptomatic with no significant medical history. He is referred for CT colonography (CTC). Which one of the following statements is correct regarding CTC?

A As much as 0.5—1% of examinations result in colonic perforation.

B A routine examination should involve supine imaging only.

C Significant extracolonic pathology is identified in 30-40% of symptom

D The administration of intravenous contrast (portal venous imaging) is advised for asymptomatic patients, as it improves the detection of colonic

E The use of an antispasmodic (eg Buscopan) immediately prior to gas insufflation enables optimal colonic distension.

A

E The use of an antispasmodic (eg Buscopan) immediately prior to gas insufflation enables optimal colonic distension.

CT colonography (CTC) is accepted as a generally safe technique with a reported perforation rate in symptomatic patients of 0.03% (compared with 0.13% with optical colonoscopy). Supine and prone imaging is widely advocated in CTC to maximise colonic distension and discriminate between faecal/ fluid bowel residue and genuine pathology, while the use of an antispasmodic is advised to avoid colonic spasm. There is no strong evidence that intravenous contrast improves detection of colonic lesions, but it may be of benefit in symptomatic patients as it enables a more accurate assessment of extracolonic pathology.

74
Q

QUESTION 52
A 71-year-old woman is referred to the on-call surgical team as an emergency admission. She complains of a 1-week history of lower abdominal pain, nausea and vomiting. She has passed loose bowel motions over the past 2 days with no bleeding per rectum. She experienced a similar, less severe, episode of lower abdominal pain 2 years ago that resolved spontaneously. On examination, the patient is pyrexial and tender in the left iliac fossa. Blood tests reveal elevated inflammatory markers and white cell count. What would be the most likely findings on a contrast-enhanced CT?

A A thickened segment of sigmoid colon with mesenteric stranding and a small pericolonic fluid collection

B Annular thickening of the sigmoid colon with several enlarged local lymph nodes

C Areas of wall thickening throughout the colon with fistulous tracts between bowel loops

D Extensive pneumoperitoneum

E Extensive wall thickening of the rectum, sigmoid and descending colon with minimal pericolonic stranding

A

A A thickened segment of sigmoid colon with mesenteric stranding and a small pericolonic fluid collection

The history is typical of acute diverticulitis. These CT findings would be consistent with ‘moderate’ diverticulitis with a small pericolonic abscess.

75
Q

QUESTION 54
A 72-year-old man is referred to hospital as an emergency admission by his GP. He has experienced vomiting and abdominal pain for 24 hours following a takeaway meal. There is a past medical history of ischaemic heart disease, chronic obstructive pulmonary disease and hypertension. An abdominal radiograph is performed and demonstrates several gas-filled loops of small bowel centrally measuring up to 2.5 cm diameter. In the left side of the abdomen, multiple round foci of gas are projected over the wall of a loop of large bowel. No free gas or mucosal thickening is identified. What is the most likely explanation for the clinical and radiographic findings?

A Gastroenteritis with incidental pneumatosis coli

B Emphysematous pyelonephritis with a paralytic ileus

C Ischaemic colitis causing intramural bowel gas

D Perforated sigmoid diverticulitis with gas in the retroperitoneum

E Small bowel obstruction due to a gallstone ileus

A

A Gastroenteritis with incidental pneumatosis coli

The presence of cyst-like gas pockets in the left hemicolon of a patientwith COPD is suggestive of pneumatosis cystoides intestinalis (pneumatosis coli).

76
Q

QUESTION 58
A 29-year-old woman received a living related bone marrow transplant for chronic myeloid leukaemia 13 days ago. She has experienced bloody diarrhoea and severe lower abdominal pain for the past 4 days and an abdominal radiograph demonstrates prominent loops of gas-filled large bowel. A contrast-enhanced CT of the abdomen is performed and shows moderate wall thickening of the right hemicolon and terminal ileum with mesenteric fat stranding. There is no abdominal lymphadenopathy and the rectum and sigmoid colon appear normal. What is the most likely diagnosis?

A Crohn’s disease

B Cytomegalovirus (CMV) colitis

C Neutropenic colitis

D PTLD

E Pseudomembranous colitis

A

C Neutropenic colitis

The patient is highly likely to be severely neutropenic at this stage and the CT findings are typical of neutropenic colitis.

77
Q

QUESTION 60
A 32-year-old man with Crohn’s disease reports increased perianal pain and swelling over a 2-month period. On examination, there is a small perineal sinus lying at die 3 o’clock position in relation to the anus. On MRI, a fistulous track of high T2 signal is seen to pass from the anal canal, through the internal sphincter and then runs medial to the external sphincter. The track reaches the skin surface of the perineum and correlates with the sinus opening on physical examination. Which description best describes this anal fistula?

A Extrasphincteric

B Infrasphincteric

C Intersphincteric

D Suprasphincteric

E Trans-sphincteric

A

C Intersphincteric

The Parks’ classification defines anal fistulae by the structures involved. The intersphincteric fistula is the most common of anal fistulae (around 70%) and does not pass through the external sphincter. A trans-sphincteric fistula will cross both the internal and external sphincters to reach the skin surface while a suprasphincteric fistula passes above the puborectalis muscle to involve the ischiorectal fossa. The least common fistula is the extrasphincteric fistula as this arises from the rectum and passes through the levator ani muscles to reach the skin surface without involving the anal sphincter mechanism at all.

78
Q

QUESTION 62
A 79-year-old man is brought to the Emergency Department with generalized abdominal pain and vomiting for 5 days. He has not opened his bowels or passed flatus during this period and has been immobile for the past 48 hours. On> examination, he is dehydrated with a distended abdomen and increased bowel sounds. An abdominal radiograph is performed and shows dilated loops of large bowel, measuring up to 5 cm in diameter. Dilated small bowel is present centrally but there is no evidence of perforation. Which statement is true regarding this clinical setting?

A Colonic pseudo-obstruction is a recognised cause of these radiographic findings.

B Diverticulitis is the most common cause of large bowel obstruction in the UK.

C Obstruction of the large bowel occurs more commonly on the right side of the colon than the left.

D Paralytic ileus is excluded by these radiographic findings.

E The ileocaecal valve is not competent in this patient.

A

A Colonic pseudo-obstruction is a recognised cause of these radiographic findings.

Colonic pseudo-obstruction can produce the same radiographic findings as large bowel obstruction: an instant contrast enema or CT should differentiate mechanical obstruction from pseudo-obstruction.

79
Q

@#e QUESTION 66
A 79-year-old woman trips and falls whilst stepping off a bus. She suffers a fractured right neck of femur and undergoes a hemiarthroplasty the following day. Her early recovery is complicated by bronchopneumonia which resolves after 5 days of broad-spectrum antibiotics. On her tenth day in hospital she develops abdominal pain and diarrhoea and pseudomembranous colitis is suspected clinically. Which one of the following statements is true regarding pseudomembranous colitis?

A A normal abdominal CT effectively excludes pseudomembranous colitis.

B Ascites is present in up to 40% of patients.

C CT carries a low positive predictive value for pseudomembranous colitis.

D Extensive pericolonic stranding is a typical feature on CT.

E The rectum is not involved in 40—50% of patients.

A

B Ascites is present in up to 40% of patients.

Ascites can occur with other colitides, but is often seen in pseudomembranous colitis. CT typically demonstrates mucosal enhancement and marked colonic wall thickening but only mild pericolonic stranding, in patients with pseudomembranous colitis. These findings have a high positive predictive value but a normal CT does not exclude pseudomembranous colitis. Rectal sparing occurs in around 10% of patients.

80
Q

QUESTION 69
A 51-year-old man attends the Emergency Department with a 3-hour history of sudden onset lower abdominal pain and vomiting. On examination, there is tenderness in the right iliac fossa but laboratory investigations are remarkable only for a mildly elevated CRP level. A contrast-enhanced CT of the abdomen is performed and demonstrates an ovoid mass lying medial to the caecum with high attenuation stranding in the pericolic fat and a central area of low density (-150 HU). There is no colonic wall thickening identified. What is the diagnosis?

A Acute appendicitis

B Epiploic appendagitis

C Ileocaecal tuberculosis

D Meckel’s diverticulum

E Right-sided diverticulitis

A

B Epiploic appendagitis

Torsion of an epiploic appendage leads to sudden onset of localised pain with characteristic CT findings of a pericolic inflammatory mass with central fat density.

81
Q

QUESTION 73
A 40-year-old woman has a 15-year history of ulcerative colitis (UC). After the initial diagnosis, she suffered frequent exacerbations of colitis requiring several hospital admissions. She declined surgical intervention at that stage and has subsequently been well controlled on medical management. Recently, she has developed a change in bowel habit and a double contrast barium enema is performed. This shows a stricture in the descending colon. Which one statement is true regarding strictures in ulcerative colitis?

A Abrupt shouldering is typical of a benign stricture in UC.

B In patients with UC, colorectal carcinomas typically arise from tubular adenomas.

C The majority of strictures in UC are benign.

D There is no increased risk of colorectal carcinoma in patients with UC.

E Widening of the presacral space is pathognomonic of a rectal carcinoma.

A

C The majority of strictures in UC are benign.

Benign strictures in UC are typically smooth and symmetrical and are due to chronic smooth muscle hypertrophy. These occur in 10—20% of patients with UC and are most common in the left colon. Carcinomas arise from dysplastic changes within diseased epithelium and not from adenomas as in the general population.

82
Q

@#e QUESTION 77
A 48-year-old man has a strong family history of colorectal cancer. He is found to have a mild microcytic anaemia and a stool sample for faecal occult blood testing is positive. A CT colonography is performed and, on 3D images, a 1-cm focal polypoid mass is seen in the wall of the sigmoid colon. The reporting radiologist is unsure whether this lesion is significant and reviews the 2D supine and prone axial images. Which additional feature would be most consistent with a polyp?

A The lesion contains a locule of gas at its base.

B The lesion has a mean density of -150 HU.

C The lesion is of homogeneous attenuation.

D The lesion lies on the dependent surface of the bowel on prone and supine images.

E There are diverticulae seen in the sigmoid colon.

A

C The lesion is of homogeneous attenuation.

A polyp will usually demonstrate uniform soft tissue density, similar to the surrounding bowel wall.

83
Q

QUESTION 85
A 79-year-old woman is admitted to hospital with a 2-day history of diarrhea and abdominal pain. A contrast-enhanced CT of the abdomen demonstrates mucosal thickening of the proximal descending colon with a low attenuation ‘target sign’ appearance. The rectosigmoid and right hemicolonare normal in appearance. The patient is managed conservatively and the symptoms resolve. Six months later, a double contrast barium enema is performed and shows an irregular stricture of the descending colon with barium sacculation. What was the original diagnosis?

A Acute diverticulitis

B Giardiasis

C Ischaemic colitis

D Pseudomembranous colitis

E Ulcerative colitis

A

C Ischaemic colitis

Stricture formation (with barium sacculation on double contrast enema) can occur in the splenic flexure due to fibrosis of the ischaemic bowel.

84
Q

QUESTION 95
A 24-year-old man is referred to the gastroenterology outpatient clinic. He describes intermittent bloody diarrhoea with abdominal pain and has lost 5 kg in weight over the past 6 months. His father and uncle both have inflammatory bowel disease. Routine laboratory investigations are remarkable for a moderately elevated CRP. A double contrast barium enema examination is performed. Which of the following findings would be more consistent with Crohn’s disease than ulcerative colitis?

A Aphthous ulceration interspersed with areas of normal mucosa

B Fine granular appearance of the descending and sigmoid colon

C Isolated involvement of the rectum and sigmoid

D Shortening and narrowing of the entire colon with absence of haustral folds

E The presence of ‘collar button’ ulceration

A

A Aphthous ulceration interspersed with areas of normal mucosa

Aphthous ulceration is the earliest sign of Crohn’s disease on a double contrast barium enema. The other options are all true of ulcerative colitis from the earliest signs of fine mucosal granularity to the ‘lead pipe’ appearance of the colon in chronic UC. Submucosal ulceration can extend laterally in UC giving the ‘collar button’ appearance.

85
Q

QUESTION 97
An 83-year-old man undergoes an emergency left hip hemiarthroplasty following a fractured neck of femur. Six days after surgery, he develops increasing abdominal distension with nausea and vomiting. An abdominal radiograph is performed and demonstrates dilatation of the ascending and transverse colon with the caecum measuring 7.0 cm in diameter. The clinical team believe that the patient may have colonic pseudo-obstruction and a single contrast (instant) enema is performed using water-soluble contrast. What are the likely findings in colonic pseudo-obstruction?

A Extrinsic compression of the sigmoid colon

B Long, irregular stricture of the sigmoid colon

C Long, smooth stricture at the splenic flexure

D No stricture demonstrated

E Short ‘apple core’ stricture of the descending colon

A

D No stricture demonstrated

An instant enema can exclude mechanical obstruction in patients with colonic pseudo-obstruction.

86
Q
  1. A 26-year-old female presents with a 1-day history of right iliac fossa (RIF) pain. She is mid-cycle and prone to mittleschmerz-type pain, but reports that this pain is more severe than previously. Serum inflammatory markers are elevated. Clinical examination reveals tenderness in the RIF, but no rebound. Due to the compounding gynaecological history, a CT is requested. This reveals a thickened caecum and thickened appendix, which appears to have a defect in the wall on the multiplanar reformatted images. There is a calcified density present in the orifice of the appendix. There is a loculated fluid collection adjacent to the appendix, which has air bubbles withinit. There is also fluid in the pelvis. A perforated appendix is removed atsurgery. Which of CT findings is most specific for perforated appendix?

A. Presence of a faecolith.

B. Identification of a wall defect.

C. Fluid in the pelvis.

D. Adjacent abscess formation.

E. Enlarged regional lymph nodes.

A
  1. D. Adjacent abscess formation.

Abscess formation has been found to be the most specific finding in appendiceal perforation, along with extraluminal gas and small bowel ileus. Abscess formation is also one of the least sensitive findings. Regional mesenteric lymph nodes are the most sensitive, but are reasonably non-specific. A focal wall defect, if seen, is reasonably sensitive and specific. Appendicolith is only found in 50% on CT and has a specificity of 70%.

87
Q
  1. A 35-year-old male patient from the Indian subcontinent presents with a2-month history of lower abdominal pain, per rectum (PR) bleeding, and weight loss. His haemoglobin is 9.4 and C-reactive protein (CRP) is 123. The patient is tender in the RIF. A CT scan is performed due the suspicion of appendiceal pathology, but unusual history. This shows bowel wall thickening of the terminal ileum with mild proximal bowel dilatation. The inner bowel wall is hypodense with enhancement of the outer bowel wall. There is stranding in the fat, which causes mass effect displacing other loops of bowel. Mild regional adenopathy is noted. The appendix is not visualized, but the caecum appears normal. There is a similar area of bowel wall thickening in the sigmoid colon. What is the most likely diagnosis?

A. Yersinia.

B. Tuberculosis.

C. Lymphoma.

D. Crohn’s disease.

E. Carcinoid.

A
  1. D. Crohn’s disease.

The findings described are classical for Crohn’s disease and lymphadenopathy is seen in up to 30% of cases. Tuberculosis more typically involves the caecum. Lymphoma usually causes a nodular appearance to the bowel. It is not associated with stricturing of the affected segment and is more classically associated with dilatation of the affected segment due to destruction of the myenteric plexus.

88
Q
  1. An overweight 42-year-old man decides to join a gym as a New Year’s resolution. During a vigorous work-out, he develops acute left lower quadrant pain and tenderness. An initial ultrasound demonstrates a small, 2-cm solid hyperechoic, non-compressible oval mass at the site of maximal tenderness. Further investigation via CT shows a pericolic pedunculated mass with fat attenuation and a hyperattenuating peripheral rim with adjacent fat-stranding abutting the anterior sigmoid colon. What is the most likely diagnosis?

A. Diverticulitis.

B. Appendicitis.

C. Epiploic appendagitis.

D. Omental infarction.

E. Sclerosing mesenteritis.

A
  1. C. Epiploic appendagitis.

Acute epiploic appendagitis is a self-limiting inflammation of the appendices epiploicae, associated with obesity, hernia, and unaccustomed exercise.
Omental infarction typically presents with pain of several days’ duration. CT demonstrates a large non-enhancing omental mass with heterogenous attenuation, and is typically located towards the right lower quadrant. Sclerosing mesenteritis is commonly located at the root of the small bowel mesentery.

89
Q
  1. A 65-year-old man, being investigated for iron deficiency anaemia, altered bowel habit, and weight loss, is diagnosed with colon cancer. A staging CT demonstrates irregularity to the outer bowel wall at the site of tumour, a cluster of three lymph nodes with the largest individual node measuring 0.9cm, and no evidence of distant metastases. What is the most likely TNM stage?

A. T2, N0, M0.

B. T2, N1, M0.

C. T3, N0, M0.

D. T3, N1, M0.

E. T4, N1, M0.

A
  1. D. T3, N1, M0.

T3 implies invasion beyond the muscularis propria into the pericolic fat. CT is quoted as 80% accurate in detecting extramural spread and nodal stage. T1 and T2 tumours are well-defined with no extension beyond the bowel contour. CT features of a T4 tumour include nodular penetration of the tumour through the peritonealized areas of the muscle coat or an advancing edge of the tumour penetrating adjacent organs. Nodal positivity is based on any node >1 cm or a group of three or more nodes.

90
Q
  1. A 64-year-old man presents to A&E with onset of severe watery diarrhea and abdominal pain. An AXR is performed which shows dilated large bowel and nodular haustral fold thickening. The patient has a CT scan with oral and intravenous contrast. The CT scan shows large bowel dilatation with diffuse bowel wall thickening. Some of the oral contrast given has become trapped between the oedematous haustral folds, causing alternating bands of high and low attenuation. What is the most likely underlying diagnosis?

A. Ulcerative colitis.

B. Crohn’s colitis.

C. Ischaemic colitis.

D. Pseudomembranous colitis.

E. Bacillary dysentery.

A
  1. D. Pseudomembranous colitis.

The CT sign of oral contrast becoming trapped between thickened oedematous haustral folds is the ‘accordion’ sign, which is highly suggestive of pseudomembranous colitis. Bowel wall thickening, pericolonic stranding of fat, and ascites may be seen in all forms of colitis. Pseudomembranous colitis can be segmental, but is more commonly a pancolitis.

91
Q

A patient with a history of inflammatory bowel disease, treated with colonic resection and J pouch anastomosis, presents to the surgical team in your hospital. The operation was 3 months ago and the initial post-operative period was unremarkable. His post-operative pouchogram was reported as normal and he underwent a reversal of his defunctioning ileostomy 6 weeks ago. He now presents with central and lower abdominal pain associated with nausea and vomiting, but no diarrhoea. The surgeons request a pouchogram, which shows a small blind ending lumen at the superior aspect of the pouch. A follow-up CT scan shows dilated small bowel with a transition point in the ileum, beyond which the bowel is non-distended. The J pouch has mild inflammatory change in the surrounding fat. There is also a small amount of free fluid in the pelvis. The wall of the pouch is not thickened. What is the most likely diagnosis?

A. Small bowel obstruction, as can occur in up to 30% of these patients.

B. Pouchitis.

C. Pouch fistula.

D. Recurrence of Crohn’s in the pouch and affected segment of bowel.

E. Pouch leak.

A

A. Small bowel obstruction, as can occur in up to 30% of these patients.

Total colonic resection with J pouch anastomosis is carried out in patients with ulcerative colitis or familial adenomatous polyposis (FAP) in order to resect the entire colon, but retain anal defaecation. One of the main contraindications to total colectomy and J pouch formation is Crohn’s disease and such a procedure is uncommonly carried out in this situation due to the high recurrence rate. Pouchitis and small bowel obstruction both occur in up to 30% of patients who undergo this procedure. The radiological features of pouchitis are non-specific, but include wall thickening and increased enhancement of the bowel wall, with peripouch fat-stranding. Fat-stranding on its own is common as a result of both the surgery and possible inflammatory change due to previous proctitis. Small bowel obstruction most commonly occurs in the region of the ileostomy. Free fluid is often seen in small bowel obstruction. The absence of fluid cavities or air pockets is against a leak from the pouch anastomosis and the clinical features are not consistent with fistula. The blind ending lumen described is the normal appearance seen on a pouchogram.

92
Q
  1. A 39-year-old male complains of severe, colicky left lower abdominal pain and rectal bleeding. He has experienced intermittent abdominal pain for the last 3–4 months. There is no previous history of medical problems. On examination he has left lower abdominal tenderness without signs of peritonism. A CT examination is performed which reveals a focal intraluminal abnormality, with the appearance of a mass within the sigmoid colon. There are concentric rings of soft tissue and fatty attenuation giving a ‘target’ like appearance. Mesenteric vessels are seen to course into the lesion. At the most distal point of the abnormality there is a more discrete low attenuation mass measuring approximately 3 cm in size of fatty attenuation. The large bowel distal to the sigmoid lesion is collapsed and proximal to the lesion there are multiple loops of small bowel and a dilated colon. What is the most likely underlying pathology for this condition?

A. Benign tumour.

B. Malignant tumour.

C. Inverted diverticulum.

D. Idiopathic.

E. Inflammatory bowel disease.

A
  1. A. Benign tumour.

The patient has presented with a colo-colic intussusception secondary to a lipoma of the colonic wall. The intussusception has been complicated by large bowel obstruction. Intussusception is caused by prolapse of a portion of the bowel into the lumen of the adjoining bowel lumen segment. Intussusception is most commonly a disease of young children and most commonly occurs in an ileocolic location. In children, over 90% of intussusceptions are idiopathic and no lead point is identified. In adult patients approximately 80% of intussusceptions are caused by lead point lesions. The CT appearances described are typical of an intussusception. The low attenuation distal lead point represents the lipoma. Ultrasound is often used to diagnose intussusception in children and will show a mass with echogenic rings representing the fat in the invaginated mesentery.

93
Q
  1. A 75-year-old man is undergoing a CT colonography examination for investigation of a change in bowel habit. He has difficulty retaining the CO2 for adequate bowel distension. Which of the following segments of colon is likely to be better distended on the prone scan?

A. Caecum.

B. Transverse colon.

C. Rectosigmoid.

D. Ascending colon.

E. Hepatic flexure.

A
  1. C. Rectosigmoid.

On a prone scan during a CT colonography examination, the rectosigmoid is generally better distended than on the supine scan because it is a more posteriorly placed structure and air gets displaced to the non-dependent position. The other segments named are usually better distended on the supine scan, particularly the caecum and transverse colon, as these are more anteriorly placed within the abdomen.

94
Q
  1. A 40-year-old male with a 22-year history of Crohn’s disease presents with abdominal pain, diarrhoea, and low-grade fever. To attempt to limit his lifetime radiation exposure he is investigated via MR enterography. Which of the following MRI findings is considered to be the earliest in active inflammation?

A. Increased mesenteric vascularity.

B. Small bowel wall thickening.

C. Mucosal hyperenhancement.

D. Perienteric inflammation.

E. Reactive adenopathy

A
  1. C. Mucosal hyperenhancement.

The lack of ionizing radiation is a major advantage to MRI for patients with Crohn’s disease given the chronic nature of this condition necessitating frequent investigation. MRI can be performed via enterography or enteroclysis. In enterography, large volumes of fluid (or a fluid inducing laxative) are ingested. Enteroclysis involves administration of enteric contrast material via a nasoenteric tube. Sequence acquistion involves fat suppression and intravenous contrast. Increased mucosal hyperenhancement (compared with that seen in normal surrounding loops) may be the earliest sign of active inflammation, even in the absence of wall thickening. Increased vascularity, perienteric inflammation, and reactive adenopathy are other signs of active Crohn’sdisease. In severe Crohn’s disease mucosal hyperenhancement combined with submucosal oedema gives a ‘stratified’ appearance. Serosal hyperenhancement may also be seen, giving a‘target’ appearance. Mural thickening is defined as greater than 3mm, although an underdistended bowel may mimic this finding.

95
Q
  1. A 20-year-old male with a recent history of medulloblastoma now presents with vague abdominal pain, PR bleeding, and weight loss. Innumerable colonic polyps are demonstrated on colonoscopy. What is the most likely unifying diagnosis?

A. Familial adenomatous polyposis.

B. Turcot syndrome.

C. Gardner syndrome.

D. Lynch syndrome.

E. Chronic inflammatory bowel disease.

A
  1. B. Turcot syndrome.

FAP is a rare autosomal dominant condition resulting in the growth of hundreds of adenomatous polyps in the large bowel. Clinical symptoms commence from the third decade and include abdominal pain and PR bleeding. Colorectal cancer develops in almost all before the age of 40years. Turcot syndrome is characterized by the association of colonic polyps similar to FAP and central nervous system tumours, typically medulloblastoma and glioblastoma multiforme. The combination of intestinal polyposis (identical to FAP) and numerous osteomas and epidermal cysts is typical of Gardner syndrome. Lynch syndrome is hereditray non-polyposis colorectal cancer. Colorectal carcinoma occurs earlier than in the average population. There is also anassociation with ovarian and endometrial malignancy. Chronic inflammatory bowel disease is not associated with CNS malignancy.

96
Q

(Ped) 74. A 15-year-old boy presents with severe abdominal pain. He has a known history of a ‘polyposis’ syndrome. A plain AXR shows small bowel obstruction. A subsequent CT scan of abdomen indicates that this is due to a small bowel intussusception. Which ‘polyposis’ syndrome does he most probably have?

A. Cronkhite–Canada syndrome.

B. Familial adenomatous polyposis syndrome.

C. Cowden disease.

D. Gardner’s syndrome.

E. Peutz–Jehger’s syndrome.

A
  1. E. Peutz–Jehger’s syndrome.

This is an autosomal dominant disease with incomplete penetrance characterized by intestinal polyposis and mucocutaneous pigmentation. It is the most frequent of the polyposis syndromes to involve the small intestine. When symptomatic, the most common presentation is abdominal pain (due to small bowel intussusception in up to 50%). There may also be GI haemorrhage. There is an increased risk of malignancy of the GI tract, pancreas, and breast, even though the polyps are hamartomas.
Cronkhite–Canada syndrome is non-neoplastic, non-hereditary inflammatory polyposis associated with ectodermal abnormalities. It usually occurs in the middle-aged to elderly and the polyps are more common in the stomach and colon than in the small bowel. Intussusception is not a typical presentation.
Gardner syndrome is an autosomal dominant disease characterized by a triad of colonic polyposis, osteomas, and soft-tissue tumours (including desmoid tumours). Age at presentation is 15–30 years. The location of the polyps is colon (100%), duodenum (90%), stomach (5–58%), and the remainder of the small bowel (<5%). Small bowel intussusception is therefore a very rare presentation. There is a very high risk of malignant transformation of the colonic polyps if left untreated. Treatment is prophylactic total colectomy.
Familial adenomatous polyposis syndrome is an autosomal dominant disease. The polyps start to appear around puberty and have a similar distribution/location to those in Gardner syndrome.
Again there is a high risk of malignant transformation of these adenomatous polyps. Cowden disease is multiple hamartoma syndrome. It causes hamartomatous neoplasms of the skin and mucosa, GI tract, bones, CNS, eyes, and genitourinary tract. There may be associated malignant tumours of the breast and thyroid.

97
Q

With regard to the use of glucagon in barium enema examinations, which is correct?

A. 0.1mg of glucagon is an appropriate dose.

B. Diabetes is a contraindication to the use of glucagon.

C. Insulinoma is a contraindication to the use of glucagon.

D. Glucagon can be safely used in patients with phaeochromocytoma.

E. Smooth muscle relaxation is optimal at 5 minutes and lasts approximately 1 hour.

A

C. Insulinoma is a contraindication to the use of glucagon.

Glucagon is a potent hypotonic agent. If 1mg of glucagon is injected intravenously it takes approximately 1 minute to work and lasts about 10–20 minutes. Intravenously administered glucagon decreases discomfort during barium enema examinations. Glucagon administration is generally safe, but is contraindicated in patients with insulinoma and phaeochromocytoma. Diabetes is not a recognized contraindication.

98
Q

A 40-year-old patient presented with right-sided lower abdominal pain. A CT was performed which showed a small pedunculated oval-shaped mass of -60HU with a more dense peripheral rim and fat stranding adjacent to the proximal descending colon. What is the most likely diagnosis?

a Acute appendicitis

b Perforated appendix with abscess

C Epiploic appendagitis

d Appendiceal tumour

e Diverticulitis

A

Answer C: Epiploic appendagitis

Epiploic appendagitis (EA) is uncommon and is due to inflammation of one of the 100 or so epiploic/omental appendages that arise from the serosal surface of the colon. Histology shows acute infarction with fat necrosis, inflammation and thrombosed vessels with haemorrhagic suffusion, but if recognised on CT it can usually be treated conservatively with spontaneous resolution within two weeks. CT appearances resolve by six months. The typical patient is a male in their forties.

99
Q

A patient with known ulcerative colitis was admitted as an emergency. Their plain abdominal radiograph showed dilation and mucosal islands in keeping with toxic megacolon. In which part of the large bowel are these changes most commonly seen?

a Caecum

b Ascending colon

C Transverse colon

d Descending colon

e Sigmoid colon

A

Answer C: Transverse colon

The transverse colon is the area of bowel where these changes are most commonly seen, probably because it is the least dependent part in the supine position. A mean dilatation of 80 mm is seen in florid cases, but anything over 55 mm is worrying. Sometimes ulceration is so extensive that no mucosal islands remain.

100
Q

A 22-year-old male with severe Crohn’s disease had recurring problems with anal fistulae. An MR fistulogram was undertaken and reported `a tract extends laterally from the rectal mucosa in the region of the dentate line through the internal sphincter and then infero-medially to skirt the medial aspect of the external sphincter and runs to an area 1 cm or so from the anus itself’. What type of fistula is this?

a Suprasphincteric

b Extrasphincteric

C Transsphincteric

d Intersphincteric

e Superficial

A

Answer D: Intersphincteric

An intersphincteric fistula passes through the internal sphincter and then between the internal and external to the perianal area.

101
Q

An 80-year-old lady presented with an acute abdomen. She suffered from diabetes, hypercholesterolaemia and had a long smoking history. On examination she has a diffusely tender abdomen with decreased bowel sounds and her pulse was irregularly irregular. Acute mesenteric ischaemia was suspected and a CT was arranged. What feature is most likely to suggest superior mesenteric artery compromise?

a Mesenteric and portal vein gas

b Small bowel pseudo-obstruction

C Gasless abdomen

d Thumbprinting of bowel wall

e Bowel distension to the splenic flexure

A

Answer E: Bowel distension to the splenic flexure

Bowel distension of the splenic flexure (i.e. the perfusion territory of the superior mesenteric artery) is seen in approximately 40-45 % of cases. The other options are all features of ischaemia but are not specific to the superior mesenteric artery territory.

102
Q

A middle-aged patient with a history of epilepsy (associated with a known supratentorial glioblastoma) underwent an elective colonoscopy and a colorectal carcinoma was identified among multiple colonic polyps that varied in size up to 30mm. Which is the most likely diagnosis?

a Familial adenomatous polyposis

b PeutzJeghers

C Turcot syndrome

d Gardner

e Cowden disease

A

Answer C: Turcot syndrome

Turcot syndrome is an autosomal recessive disease characterised by colonic polyposis and central nervous system tumours especially supratentorial glioblastoma and occasionally medulloblastomas. Malignant transformation of the polyps is expected.

103
Q

A 24 year old developed crampy abdominal pain and diarrhoea over the course of several months and lost a significant amount of weight. He was reluctant to undergo endoscopy and was referred for a barium meal which showed narrowing of the gastric antrum and a cobblestone appearance of the gastric and duodenal mucosa. What is the most likely diagnosis?

a Crohn’s disease

b Submucosal metastases

c Zollinger-Ellison syndrome

d Erosive gastritis

e Gastric carcinoma

A

Answer A: Crohn’s disease

Although Crohn’s disease more frequently affects the small bowel, involvement of the stomach and duodenum occurs in up to 10% of patients. Typical appearances include aphthoid ulcers, erosions, cobblestone mucosa and thickened duodenal folds.

104
Q

A 60-year-old male presented with change in bowel habit, weight loss and anaemia. Flexible sigmoidoscopy demonstrated a mass in the sigmoid colon and biopsy showed it to be an adenocarcinoma. MRI of the pelvis showed invasion of the muscularispropria and CT showed eight significantly enlarged local pericolic nodes. No further significant lymphadenopathy was present and there were no lung metastases. What radiological staging is most appropriate?

a T1, NO, MI

b T2, N2, M0

c T3, N2, MO

d T4, NO, MO

e T2, NI, MO

A

Answer B: T2, N2, MO.

105
Q

A 67-year-old female had a CT of her abdomen and pelvis to investigate abdominal pain. This showed a suspicious rectosigmoid lesion, which was subsequently biopsied and found to be adenocarcinoma. What would be the next appropriate imaging?

a Ultrasound liver, CXR, MRI pelvis

b MRI pelvis and CT chest

C MRI pelvis and MRI liver

d CT chest, MRI pelvis and whole body PET

e Whole body PET and MRI pelvis

A

Answer B: MRI pelvis and CT chest.

106
Q

A young female patient with Crohn’s disease was due to be assessed for recurrent episodes of perianal sepsis. What is the most appropriate modality for imaging the pelvis?

a Transvaginal ultrasound

b Contrast-enhanced CT of abdomen and pelvis

C PET-CT scan

d MRI pelvis

e Transrectal ultrasound

A

Answer D: MRI pelvis

MRI is now the modality of choice for recurrent perianal sepsis as it can delineate the anatomy and identify collections and fistulae.

107
Q

A patient with infectious colitis underwent an abdominal CT scan. Thickening and reduced attenuation of the colonic wall of the right hemicolon was visible. The remainder of the bowel was unremarkable. What causative organism is most likely to account for these radiological findings?

a Cytomegalovirus (CMV)

b Escherichia coli

C Gonorrhoea

d Clostridium diffacile

e Shigella

A

Answer E: Shigella
`
The pattern of colonic involvement can give useful clues to the causative organism in infectious colitis. Organisms that cause colitis limited to the right colon include Shigella and Salmonella.

108
Q

A 52-year-old man was admitted to the medical high dependency unit with a severe colitis. Routine biochemical, haematological and microbiological specimens were taken and a plain abdominal radiograph and portable abdominal ultrasound were performed. Further enquiry revealed that he had recently been treated with broad spectrum antibiotics, but also has a background of a longer history of colitic symptoms. What radiological features would support
a diagnosis of pseudo-membranous colitis rather than ulcerative colitis?

a Arthritis
b Ascites

C Excessive rectosigmoid fat

d Polyp formation

e Pancolitis

A

Answer B: Ascites

CT is abnormal in 60% of cases of pseudomembranous colitis. Marked colonic wall thickening is the commonest manifestation. Ascites occurs in 35%.

109
Q

An otherwise well 19-year-old male was seen in Gastroenterology Outpatients with a long history of diarrhoea and weight loss. He was noted to have mouth ulcers and a few reddened raised areas on his shins. A small bowel followthrough, colonoscopy and further blood tests were arranged. What is the most likely finding on the follow-through?

a Linear ulcers on mesenteric border

b Shallow small rounded ulcers

C Double tracking ulcers

d Single large (>5 cm) deep ulcer

e Mass lesion

A

Answer A: Linear ulcers on mesenteric border

Linear ulcers on the mesenteric border are nearly pathognomonic of Crohn’s disease. Ulcers are most commonly multiple in inflammatory bowel disease. Double tracking ulcers are usually seen in the colon in the context of ulcerative colitis.

110
Q

26 A patient was being investigated for colitis of unknown aetiology. Infective causes were excluded and both endoscopy and biopsy have proved inconclusive. What imaging feature would be more in keeping with ulcerative colitis than Crohn’s disease?

a Fistula formation

b Evidence of ileitis

c Rectal involvement

d Skip lesions

e Stricturing

A

26 Answer C: Rectal involvement

Rectal involvement is commoner in ulcerative colitis (UC) (95%) than Crohn’s (50%).

111
Q

28 A 77-year-old male was investigated for chronic, intermittent low-grade gastrointestinal bleeding. He was otherwise generally well but had had an aortic valve replacement for severe aortic stenosis. Colonoscopy was unremarkable and a Tc-99m labelled red blood cell scan was performed. This showed a focus of tracer accumulation in the region of the caecum and ascending colon. During an acute episode of bleeding mesenteric angiography was performed which showed an arterial tuft and early opacification of the ileocolic vein. What is the most likely lesion to account for these appearances?

a Angiodysplasia of the colon

b Colonic carcinoma

C True arteriovenous malformation

d Colonic polyp

e Diverticular disease

A

28 Answer A: Angiodysplasia of the colon

Angiodysplasia of the colon is the most common vascular malformation of the GI tract and is due to vascular ectasia rather than a true arteriovenous malformation (AVM). It is caused by age related degenerative dilatation and is associated with aortic stenosis in 20%. It is most commonly seen in the caecum/ascending colon.

112
Q

29 A 44-year-old female was found to have colorectal carcinoma in the transverse colon on colonoscopy. There were no other sites of disease and no colonic polyps were seen. She remembers three immediate family members from different generations who have also been diagnosed with colonic carcinoma. As she also had occasional haematuria, imaging of her renal tract was arranged which showed a transitional cell carcinoma of the renal pelvis. Which syndrome is she most likely to have?

a Familial adenomatous polyposis

b Lynch I

C Lynch II

d Gardner’s

e Turcot

A

29 Answer C: Lynch II

Lynch is otherwise known as hereditary nonpolyposis colorectal cancer syndrome. The Amsterdam criteria of this syndrome are based on relatives who have been diagnosed with colorectal cancer. (Three or more family members of whom two are first-degree relatives of the third, family members in two or more generations, one family member diagnosed <50 years old.) It is present in 5-10% of individuals with colon cancer. Lynch I has no association with extracolonic malignancies. Lynch II is associated with other malignancies including transitional cell carcinoma of the ureter and renal pelvis.

113
Q

30 A 66-year-old male with a small local rectal carcinoma was followed up with a restaging CT after six months’ treatment. He felt generally well but had a past medical history including three previous myocardial infarctions with coronary angioplasty on two occasions. He suffered from hypercholesterolaemia and had a smoking history of 69 pack years. His exercise tolerance was poor and could only walk 50 metres before becoming breathless. No local recurrence or metastatic disease was identified but there were small bubbles of air within the lamina propria of a large proportion of the large bowel and a small amount of gas was seen in the portal vein. What is the most likely cause of the bowel abnormalities?

a Bowel necrosis/gangrene

b Mucosal disruption

c Increased mucosal permeability

d Pulmonary disease

e Radiotherapy related

A

30 Answer D: Pulmonary disease

There are over 60 causative factors for pneumatosis intestinalis. However, there are four main mechanisms by which they occur: bowel necrosis/gangrene, mucosal disruption, increased mucosal permeability and pulmonary disease. As a general rule pneumatosis of the colon is likely to be clinically insignificant and the extent of the pneumatosis is inversely related to the severity of the disease. A well man is unlikely to have ischaemic bowel and with a significant smoking history and probable symptomatic lung disease then pulmonary disease is the most likely cause. He has no good history for the other two mechanisms. Gas can be seen in the mesenteric and portal veins with pulmonary disease.

114
Q

32 A patient previously diagnosed with HIV presented with abdominal pain, fever and malaise. Imaging findings showed a pancolitis, ascites and aphthous ulcers on a background of normal mucosa. What is the commonest cause of these findings?

a Herpes simplex

b Candida

C Cytomegalovirus (CMV)

d Tuberculosis

e Clostridium difficile

A

32 Answer C: Cytornegalovirus (CMV)

CMV is the commonest cause of life-threatening opportunistic infection in AIDS patients and accounts for 13 % of all gastrointestinal disease in this population.

115
Q

33 A 37-year-old male presented with sudden onset severe abdominal pain and abdominal distension. A plain abdominal radiograph showed a distended loop of large bowel that was kidney shaped. He went on to have a contrast enhanced CT of his abdomen and pelvis. What is this most likely to show?

a Appendicitis

b Small bowel obstruction

C Ischaemic bowel

d Caecal volvulus

e Sigmoid volvulus

A

33 Answer D: Caecal volvulus

Caecal volvulus accounts for 40% of all colonic volvuluses and is associated with malrotation and a long mesentery giving poor fixation of the right colon (10-25 % of the population). If caecal distension greater than 10-12 cm occurs, there is an increased risk of perforation or bowel infarction.

116
Q

34 A young patient with Crohn’s disease and recurrent episodes of perianal sepsis was found to have an enterocolic fistula. What is the most likely location of this fistula?

a Ileocolic

b Ileocaecal

C Ileo-ileal

d Colo-coloic

e Ileosigmoid

A

34 Answer B: Ileocaecal

Crohn’s disease is the third commonest cause of fistula after iatrogenic and diverticular fistulae. Enterocolic are most often between ileum and caecum; other fistula may occur: enterocutaneous (8-21 %), rectum to skin or vagina, peri-anal fistulae and sinus tracts.

117
Q

35 A patient with inflammatory bowel disease was found to have colonic polyps on CT colonography. What feature would be more consistent with a diagnosis of pseudopolyposis (postinflammatory polyposis) than true polyposis?

a Ill-defined margins

b Preserved colonic haustra

c Round

d Uniform in size

e Well-delineated

A

35 Answer A: Ill-defined margins

118
Q

36 An elderly woman who was a long-term psychiatric inpatient was referred to the duty surgical team with symptoms of large bowel obstruction. A plain abdominal radiograph showed a grossly dilated loop of large bowel and a subsequent abdominal CT demonstrated a whirled appearance of the associated mesentery. What is the most likely diagnosis?

a Caecal volvulus

b Giant sigmoid diverticulum

c Perforated sigmoid diverticulum

d Retractile mesenteritis

e Sigmoid volvulus

A

36 Answer E: Sigmoid volvulus

Sigmoid volvulus is common in elderly and psychiatric patients. The typical plain radiograph appearance is of a dilated loop of sigmoid with a distinct midline crease - the `coffee-bean’ sign. Twisting of the mesentery is seen on CT.

119
Q

28 A 32-year-old female was diagnosed with Crohn’s disease. She had ongoing problems with rectoanal fistulae and was keen to explore a surgical solution if appropriate. The last CT of her abdomen and pelvis was performed over 18 months ago and the inflammation is currently relatively quiescent. What is the most appropriate way of further investigating the fistulae?

a MR fistulography

b Contrast enema

c Transrectal ultrasound

d No imaging, surgical exploration only

e Repeat contrast-enhanced CT

A

28 Answer A: MR fistulography

120
Q

29 An elderly patient with acute lower gastrointestinal bleeding underwent mesenteric angiography with the aim of identifying and treating the source of the bleeding. Which is the usual order to performing this study?

aThe superior mesenteric artery is evaluated before the inferior mesenteric artery

b The inferior mesenteric artery is evaluated before the superior mesenteric artery

c There is no recommended order; evaluation should be based on the order in which the arteries happen to be catheterised

d The coeliac axis is evaluated before the superior mesenteric artery

e The renal arteries are evaluated first, followed by the other vessels in any order

A

29 Answer B: The inferior mesenteric artery is evaluated before the superior mesenteric artery

The inferior mesenteric artery is usually evaluated first as once the patient’s bladder begins to fill with contrast it may become more difficult to evaluate this area. The celiac axis is not routinely assessed when investigating lower gastrointestinal bleeding.

121
Q

30 A 58-year-old male with rectal carcinoma underwent staging with CT and MRI. The MRI reported `there is transmural extension of the local disease but no invasion of adjacent organs. There is a single lymph node measuring 1.2 cm in the short axis seen within the mesorectal envelope’. No distant disease was seen on the CT. On the basis of this information what is the TMN staging?

a T2 NO MO

b T2 N1 MO

c T3 NO MO

d T3 N1 MO

e T4 NI MO

A

30 Answer D: T3 N1 MO

122
Q

31 A 65-year-old male presented with severe acute abdominal pain. He had a past history of ischaemic heart disease and was in atrial fibrillation. He had diffuse abdominal pain with generalised tenderness but was not peritonitic. Blood tests showed raised inflammatory markers and blood gases demonstrated a metabolic acidosis with a raised lactate. A plain abdominal radiograph showed thumbprinting of the colon wall and a contrast-enhanced CT demonstrated colonic mucosal oedema, sparing the small bowel and rectum. What is the most likely diagnosis?

a Crohn’s disease

b Ulcerative colitis

c Ischaemic colitis

d Sigmoid volvulus

e Pancreatitis

A

31 Answer C: Ischaemic colitis

The commonest cause of acute ischaemic bowel is an occlusive superior mesenteric artery (SMA) embolus - in more than 50% of cases. Other causes are non-occlusive thrombus, SMA dissection and venous occlusion.

123
Q

32 A 58-year-old male presented with intermittent low-grade rectal bleeding. Angiography showed a cluster of vessels on the anti-mesenteric border in the caecal region in arterial phase with early opacification of an ileo-colic vein and a densely opacified tortuous ileo-colic vein in the late venous phase. What is the most likely diagnosis?

a Meckel’s diverticulum

b Angiodysplasia

c Crohn’s disease

d Caecal ulcer

e Caecal carcinoma

A

32 Answer B: Angiodysplasia

This is due to age-related degeneration and dilatation of the submucosal vessels in the bowel wall and is the commonest vascular lesion of the GI tract. The caecum and ascending colon are most commonly affected followed by the descending and sigmoid colon. Less frequently, the small bowel may also be affected.

124
Q

33 An elderly depressed patient was admitted from his nursing home on numerous occasions with abdominal pain and distension. On each occasion a plain abdominal radiograph was performed which showed multiple distended fluid-filled loops of bowel with the appearance of a `coffee bean’. What is the most likely diagnosis?

a Appendicitis

b Small bowel obstruction

c Ischaemic bowel

d Caecal volvulus

e Sigmoid volvulus

A

33 Answer E: Sigmoid volvulus

This is commoner in the elderly and institutionalised populations. Varying degrees of torsion can occur, the commonest being through 360 degrees.

125
Q

34 A patient underwent a barium enema as an investigation for change in bowel habit. Apart from a few diverticula the only positive finding was widening of the pre-sacral space. In 95 % of cases, what is the accepted value for a normal pre-sacral space?

a >5 mm

b <5 mm

C >10mm

d 15 mm

e > 15 mm

A

34 Answer B: <5 mm

The normal pre-sacral space in adults in 95 % of cases is <5 mm and abnormal is > 10 mm.

126
Q

35 An 80-year-old female presented with a history of passing blood and mucus per rectum. She had no history of weight loss or inflammatory bowel disease and no family history of colorectal cancer. She was unable to tolerate bowel preparation and was therefore referred for a minimally prepared CT colon. A polyp was visible in her descending colon. What further finding would most suggest this polyp was malignant?

a 1.5 cm in size

b Lobulation

c Pedunculated

d Solitary

e Smooth underlying colonic wall

A

35 Answer B: Lobulation

127
Q

36 A young HIV-positive man presented with localised peritonism and was further assessed with CT He had recently had a positive stool culture for Salmonella, which was felt to be responsible for his symptoms. Which part of the bowel is most likely to appear abnormal on CT?

a Terminal ileum

b Right hemicolon

c Left hemicolon

d Sigmoid

e Entire colon

A

36 Answer B: Right hemicolon

128
Q

An 18-year-old girl presented with a one-year history of recurrent colicky abdominal pain, diarrhoea, anorexia and weight loss. Blood tests showed an iron deficiency anaemia. Gastroscopy and colonoscopy as far as the hepatic flexure were normal. A small bowel followthrough showed thickened ileal folds, aphthous ulcers, cobblestoning and terminal ilealstricturing. What is the most likely diagnosis?

a Crohn’s disease

b Ulcerative colitis

c Bowel ischaemia

d Appendicitis

e Mesenteric venous thrombosis

A

Answer A: Crohn’s disease

The age of presentation of Crohn’s disease is typically 15-30 years with an equal sex distribution. The symptoms are often vague and may be present for some time before the diagnosis is made. It can affect any part of the GI tract from the mouth to the anus and can skip segments.

129
Q

63 A young female was admitted with generalised lower abdominal pain. She had a mild pyrexia and borderline raised inflammatory markers. Appendicitis was considered as a diagnosis and an ultrasound was requested. What ultrasound finding would be most supportive of this?

a Compressible tubular structure >4mm thick

b Non-compressible tubular structure >4mm thick

C Compressible tubular structure >6 mm thick

d Non-compressible tubular structure >6 mm thick

e Wall thickness of 1 mm

A

63 Answer D: Non-compressible tubular structure >6 mm thick

Ultrasound features of appendicitis are total thickness >6 mm, non-compressibility, wall thickness >3 mm and a shadowing appendicolith.

130
Q
  1. A 35-year-old man with history of multiple scalp lesions and dental caries presents with abdominal pain and vomiting. CT of the abdomen shows extensive colonic lesions and a duodenal mass causing partial obstruction.
    The most likely diagnosis is?

(a) Familial adenomatous polyposis

(b) Lymphoma

(c) Inflammatory polyps

(d) Ulcerative colitis

(e) Whipple’s disease

A
  1. (a) Familial adenomatous polyposis

A combination of extensive colonic polyposis with extracolonic features such as osteoma of the skull and mandible, epidermoid cysts, abnormal dentition and desmoid tumours is called Gardner’s syndrome and is now included as a part of the spectrum of familial adenomatous polyposis.

131
Q
  1. A 40-year-old man presented with left lower abdominal pain. A CT scan demonstrates a 2 cm fat density lesion surrounded by a hyperdense rim and inflammatory fat stranding abutting the sigmoid colon although no wall thickening or sigmoid diverticulosis was seen. The most likely diagnosis is?

(a) Appendicitis

(b) Epiploic appendagitis

(c) Diverticulitis

(d) Mesenteric panniculitis

(e) Omental infarction

A

12.(b) Epiploic appendagitis

The typical appearance of this is a 2–4 cm fatty lesion surrounded by a rim of tissue and inflammatory change next to colon.
An important differential is acute diverticulitis, which commonly has a background of diverticulosis and colonic wall thickening

132
Q
  1. A 30-year-old man presents with weight loss and diarrhoea. There is family history of father having total colectomy at age of 21 years. On colonoscopy the patient is found to have hundreds of polyps in the colon. CT shows a enhancing mass in the second part of duodenum. The most likely diagnosis is?

(a) Juvenile polyposis

(b) Familial adenomatous polyposis

(c) Metaplastic polyposis

(d) Peutz–Jeghers syndrome

(e) Turcot’s syndrome

A
  1. (b) Familial adenomatous polyposis

By definition, hundreds of polyps have to be present for this diagnosis. It is an autosomal dominant condition and the polyps present around puberty. Almost all patients have duodenal adenomas, with a 5% risk of conversion to periampullary carcinoma. Juvenile polyposis is rare and presents in infancy. Multiple metaplastic polyps are very rare. Peutz–Jeghers syndrome shows few colonic polyps, and most are seen in the small bowel. Turcot’s syndrome is a rare association between colonic carcinoma and medulloblastoma.

133
Q
  1. A 60-year-old man with history of aortic valve replacement presents with bleeding per rectum. Colonoscopy is normal. On arteriography, a cluster of vessels are seen on the antimesenteric border of the ascending colon, with early opacification of the ileocolic vein. The most likely diagnosis is?

(a) Occult carcinoma

(b) Angiodysplasia

(c) Gastrointestinal stromal tumour

(d) Portal hypertension

(e) Diverticulitis

A
  1. (b) Angiodysplasia

Angiodysplasia is associated with aortic stenosis and commonly found in the ascending colon. This is seen at the antimesenteric border. On angiography, there is cluster of vessels seen on arterial phase with early filling of the draining vessel. These can be small lesions and may be missed on colonoscopy.

134
Q
  1. A 60-year-old man presents with constipation and painful defecation. CT shows a non-enhancing, well-defined, lobulated, and homogenous, low-attenuation lesion in the retrorectal space. The lesion shows thin peripheral calcification. On MRI, the lesion returns intermediate signal on T1 with areas of high signal within and high signal on T2 with septae. What is the most likely diagnosis?

(a) Enteric cyst

(b) Dermoid cyst

(c) Sacrococcygeal teratoma

(d) Anal duct cyst

(e) Rectal leiomyosarcoma

A
  1. (a) Enteric cyst

Enteric cyst may be septated and filled with mucoid contents which return high signal on T1 images. On CT they have the characteristic features as in the case. Dermoid cysts contain skin appendages and commonly contain fat. Sacrococcygeal teratomas are usually seen in paediatric age group. On CT and MRI, they appear as heterogeneously enhancing lesions with both cystic and solid components.

135
Q
  1. A 40-year-old man presents with a 4-week history of right iliac fossa pain. Ultrasound shows a tubular fluid filed lesion in the right lower abdomen. CT shows a homogenous prominent appendix of high attenuation (30 Hounsfield units). No inflammatory change, calcifications, lymph nodes or free fluid are seen. The most likely diagnosis is?

(a) Normal appendix

(b) Lymphoma of the appendix

(c) Mucocoele of the appendix

(d) Carcinoma of the appendix

(e) Carcinoid of the appendix

A
  1. (a) Mucocoele of the appendix

A mucocoele is a chronic cystic dilatation of the appendiceal lumen caused by mucin collection. On ultrasound it is anechoic or hypoechoic, depending on the composition of mucous. On CT the lesion is homogenous and cystic. On MRI a mucocoele is hyperintense on T2 and hypo-isointense on T1.

136
Q
  1. A 40-year-old patient with acute myeloid leukaemia on chemotherapy presents with acute-onset pain in the right lower abdomen and diarrhoea. CT shows circumferential thickening of the caecal wall and mild surrounding inflammatory changes in the mesenteric fat. What is the most likely diagnosis?

(a) Epiploic appendagitis

(b) Typhlitis

(c) Ulcerative colitis

(d) Crohn’s disease

(e) Ischaemic colitis

A
  1. (b) Typhlitis

Given the history of acute myeloid leukaemia with chemotherapy, this is the most likely diagnosis

137
Q
  1. A 50-year-old woman with a history of bleeding per rectum presents for a barium enema. The examination shows multiple worm-like projections attached by their bases to the sigmoid colon. The most likely diagnosis is?

(a) Postinflammatory polyps

(b) Juvenile polyps

(c) Familial adenomatous polyposis

(d) Faecal residue

(e) Colonic ulceration

A
  1. (a) Postinflammatory polyps

Filiform polyps adhering to the colon by their bases are typical of postinflammatorypolyps, which can be seen in ulcerative colitis.

138
Q
  1. A 70-year-old man with history of CVA and coronary artery bypass grafting presents with bloody diarrhoea, abdominal pain and vomiting. Blood gases show severe metabolic acidosis. CT abdomen shows focal area of thickened sigmoid flexure with pericolic stranding and normal right colon. The most likely diagnosis is?

(a) Ulcerative colitis

(b) Ischaemic colitis

(c) Infection

(d) Crohn’s disease

(e) Pseudomembranous colitis

A
  1. (b) Ischaemic colitis

Ischaemic colitis is usually due to indolent non-occlusive atherosclerosis in the elderly. Colonic wall thickening in a watershed area suggests ischaemia as the aetiology. Risk factors include cardiovascular disease, diabetes, vasculitis, etc. On CT, there is colonic wall thickening (> 5 mm), in a watershed distribution. The colon wall may appear non-enhancing compared with normal colon or it may appear dense due to haemorrhage. Pneumatosis coli, portal vein gas and perforation may be found.

139
Q
  1. Features of pseudomembranous colitis include:

(a) An acute infective colitis due to Chostridium perfringens toxin.

(b) Most commonly affects the transverse colon.

(c) Bowel wall thickening is the commonest appearance on non-contrast CT images.

(d) ‘Thumbprinting’ is seen on the plain abdominal radiograph.

(e) Ascites is a recongnised feature.

A

Answers:

(a) Not correct
(b) Not correct
(c) Correct
(d) Correct
(e) Correct

Explanation:

Pseudomembranous colitis is caused by Clostridium difficile toxin. It most commonly affects the rectum. Ascites is a recognised feature in severe cases.

140
Q
  1. Regarding ischaemic colitis:

(a) Griffith point is the most commonly affected segment.

(b) The right colon is involved in 30% of cases.

(c) Usually occurs in the first decade of life.

(d) Barium enema is usually only abnormal in 50-60% of cases.

(e) Portal vein gas is of little clinical significance.

A

Answers:

(a) Correct
(b) Correct
(c) Not correct
(d) Not correct
(e) Not correct

Explanation:

Ischaemic colitis is usually seen in patients >50 years of age. Barium enema is abnormal in 90% of the cases showing features of bowel wall thickening, loss of haustrations and thumbprinting. Evidence of portal vein gas is seen in very rae cases and is a preterminal sign.

141
Q

@#e 11. Regarding diverticular disease:

(a) Colonic diverticulosis affects 70-80% by 80 years of age.

(b) Rectosigmoid colon is most commonly affected.

(c) 10-25% of individuals with colonic diverticular disease develop diverticulitis.

(d) Fistula formation occurs in 40-50% of cases complicating acute diverticulitis.

(e) Moderate diverticulitis is present when the bowel wall is thickened >3mm.

A

Answers:

(a) Correct
(b) Correct
(c) Correct
(d) Not correct
(e) Correct

Explanation:

Fistula formation is seen in 15% of the cases of complicated acute diverticulitis.

142
Q
  1. Which of the following are correct about Crohn’s disease:

(a) There is an increased risk of malignancy.

(b) Pseudo-diverticula are typically found on the antimesenteric side of the bowel.

(c) Mural stratification on CT indicates active disease.

(d) Colonic involvement is usually accompanied by small bowel disease.

(e) Apthous ulcers are an early finding.

A

Answers:

(a) Correct
(b) Correct
(c) Correct
(d) Correct
(e) Correct

Explanation:

The comb sign in crohn’s disease depicts the pericolic and perienteric fat stranding due to increased mesenteric vasculature. Disease can affect any part of the gastrointestinal tract from mouth to the anus however small intestine is most frequently involved particularly the terminal ileum.

143
Q
  1. In the imaging of acute colitis, which of the following are correct?

(a) Paucity of pericolonic inflammation is more suggestive of pseudomembranous colitis than ulcerative colitis.

(b) Toxic megacolon is not a feature of pseudomembranous colitis.

(c) In pseudomembranous colitis, the most common site involved is the rectosigmoid.

(d) Portal venous gas is a more specific sign of ischaemic colitis than pneumatosis cystoides intestinalis.

(e) Neutropenic colitis (typhlitic colitis) most commonly affects the descending and sigmoid colon.

A

Answers:

(a) Correct
(b) Not correct
(c) Not correct
(d) Correct
(e) Not correct

Explanation:

CT features of pseudomembranous colitis are non-specific and include mural thickening with bowel dilatation. There is pancolitis & right-sided colitis. The rectosigmoid is spared in 67% of the cases and ascites is not uncommon. Complications include toxic megacolon on operation and peritonitis. Typhlitis is seen in neutropenia patients and usually presents as non-specific thickening of caecum and ascending colon due to necrosis.